Medsurg2 Success Ch 7 GI

Réussis tes devoirs et examens dès maintenant avec Quizwiz!

The nurse at the scene of a knife fight is caring for a young man who has a knife stuck in his abdomen. Which action should the nurse implement? 1. Stabilize the knife. 2. Remove the knife gently. 3. Turn the client on the side. 4. Apply pressure to the insertion site.

**1. Do not remove any penetrating object in the abdomen; removal could cause further internal damage. 2. Removal of the knife could cause further internal damage. 3. The client should be kept on the back and the knife should be stabilized. 4. The nurse should stabilize the knife and get the client to the emergency department; pressure could increase abdominal damage.

Which intervention should the nurse implement when administering a potassium supplement? 1. Determine the client's allergies. 2. Assess the client's apical heart rate. 3. Monitor the client's blood pressure. 4. Monitor the client's complete blood count.

1. The nurse should inquire about drug allergies before administering all medications, not just potassium. **2. Cardiac dysrhythmias occur when serum potassium levels are too low or too high. Many dysrhythmias can be detected by assessing the regularity of the heart rate. 3. The blood pressure does not evaluate for dysrhythmias, a possible result of abnormal potassium levels. 4. The complete blood count does not include the potassium level; a chemistry panel would be needed.

The nurse is teaching the client diagnosed with inflammatory bowel disease (IBD) about the therapeutic diet. Which food selection would be the best choice for a meal? 1. Roast beef on wheat bread and a milk shake. 2. Hamburger, French fries, and a Coke. 3. Pepper steak, brown rice, and iced tea. 4. Roasted turkey, asparagus, and water.

33. 1. Wheat bread and all whole grains should be avoided, and most clients cannot tolerate milk products. 2. Fried foods such as hamburger and French fries should be avoided. Raw fruits and vegetables such as lettuce and tomatoes are usually not tolerated. 3. Whole grains such as brown rice should be avoided. White rice can be eaten. Spicy meats and foods should be avoided. **4. Meats can be eaten if prepared by roasting, baking, or broiling. Vegetables should be cooked, not raw, and skins should be removed. A low-residue diet should be eaten.

The client is admitted into the emergency department complaining of acute epigastric pain and reports vomiting a large amount of bright-red blood at home. Which interventions should the nurse implement? List in order of priority. 1. Assess the client's vital signs. 2. Insert a nasogastric tube. 3. Begin iced saline lavage. 4. Start an IV with an 18-gauge needle. 5. Type and crossmatch for a blood transfusion.

In order of priority: 1, 4, 5, 2, and 3 1. The nurse should assess the vital signs to determine if the client is in hypovolemic shock. 4. The nurse should start the IV line to replace fluid volume. 5. While the nurse is starting the IV, a blood sample for typing and cross-matching should be obtained and sent to the laboratory. 2. An N/G tube should be inserted so that direct iced saline can be instilled to cause constriction, which will decrease the bleeding. 3. The iced saline lavage will help decrease bleeding.

The nurse is administering a proton pump inhibitor to the client diagnosed with peptic ulcer disease. Which statement supports the rationale for administering this medication? 1. It prevents the final transport of hydrogen ions into the gastric lumen. 2. It blocks receptors that control hydrochloric acid secretion by the parietal cells. 3. It protects the ulcer from the destructive action of the digestive enzyme pepsin. 4. It neutralizes the hydrochloric acid secreted by the stomach.

**1. This is the rationale for proton pump inhibitors. 2. This is the rationale for histamine receptor antagonists. 3. This is the rationale for mucosal protective agents. 4. This is the rationale for antacids.

The client has recently been diagnosed with irritable bowel syndrome (IBS). Which intervention should the nurse teach the client to reduce symptoms? 1. Instruct the client to avoid drinking fluids with meals. 2. Explain the need to decrease intake of flatus-forming foods. 3. Teach the client how to perform gentle perianal care. 4. Encourage the client to see a psychologist.

**1. This will help prevent abdominal distention, which causes symptoms of IBS. Do not confuse inflammatory bowel disease (IBD) and irritable bowel syndrome (IBS). 2. Flatus (gas) forming foods will help prevent symptoms of IBD, not IBS. 3. Clients with IBS do have altered bowel habits such as diarrhea and constipation, but perianal care will not prevent IBS. 4. IBS does have a psychological component, but a client recently diagnosed should be taught other interventions before a psychologist is recommended.

The client diagnosed with AIDS is experiencing voluminous diarrhea. Which interventions should the nurse implement? Select all that apply. 1. Monitor diarrhea, charting amount, character, and consistency. 2. Assess the client's tissue turgor every day. 3. Encourage the client to drink carbonated soft drinks. 4. Weigh the client daily in the same clothes and at the same time. 5. Assist the client with a warm sitz bath PRN.

***1. It is important to keep track of the amounts, color, and other characteristics of all body fluids lost. 2. Skin turgor should be assessed at least every six (6) to eight (8) hours, not daily. 3. Carbonated soft drinks increase flatus in the GI tract, and the increased sugar will act as an osmotic laxative and increase the diarrhea. ***4. Daily weights are the best method of determining fluid loss and gain. ***5. Sitz baths will assist in keeping the client's perianal area clean without having to rub. The warm water is soothing, providing comfort. TEST-TAKING HINT: The test taker should note the time frame for any answer option. Every day is not often enough to assess for dehydration in a client who is experiencing massive (voluminous) fluid loss. If the test taker were not aware of the definition, then an associated word "volume" would be a hint.

The nurse is caring for clients on a medical unit. Which client information should be brought to the attention of the HCP immediately? 1. A serum sodium of 139 mEq/L in a client diagnosed with obstipation. 2. The client diagnosed with fecal impaction who had two (2) hard formed stools. 3. A serum potassium level of 3.0 mEq/L in a client diagnosed with diarrhea. 4. The client with diarrhea who had two (2) semi-liquid stools totaling 300 mL.

***1. Normal serum sodium levels are 135-152 mEq/L, so this falls within the normal range. 2. The client diagnosed with a fecal impaction is beginning to move the stool; this indicates an improvement. 3. Normal potassium levels are 3.5-5.5 mEq/L. The level stated in this option is below normal. Imbalances in potassium levels can be caused by diarrhea and can cause cardiac dysrhythmias. 4. This client has been having diarrhea and now is having semi-liquid stools, so this client is getting better. TEST-TAKING HINT: The test taker must determine if the client is experiencing a potentially life-threatening complication, such as potential for cardiac dysrhythmias. Answer options "1," "2," and "4" are expected for the disease process and are normal or show improvement.

The client who has had an abdominal surgery has a Jackson Pratt (JP) drainage tube. Which assessment data would warrant immediate intervention by the nurse? 1. The bulb is round and has 40 mL of fluid. 2. The drainage tube is pinned to the dressing. 3. The JP insertion site is pink and has no drainage. 4. The JP bulb has suction and is sunken in.

***1. The JP bulb should be depressed, which indicates suction is being applied. A round bulb indicates that the bulb is full and needs to be emptied and suction reapplied. 2. The tube should be pinned to the dressing to prevent accidentally pulling the drain out of the insertion site. 3. The insertion site should be pink and without any signs of infection, which include drainage, warmth, and redness. 4. The JP bulb should be sunken in or depressed, indicating that suction is being applied. TEST-TAKING HINT: The stem is asking which data need intervention by the nurse. Option "2" can be ruled out because all tubes and drains should be secured. A pink insertion site with no drainage is expected, which would cause the test taker to eliminate this option as a possible correct answer.

The client presents to the emergency department experiencing frequent watery, bloody stools after eating some undercooked meat at a fast food restaurant. Which intervention should be implemented first? 1. Provide the client with a specimen collection hat to collect a stool sample. 2. Initiate antibiotic therapy intravenously. 3. Have the laboratory draw a complete blood count. 4. Administer the antidiarrheal medication Lomotil.

***1. This client may have developed an infection from the undercooked meat. The nurse should try to get a specimen for the laboratory to analyze and for the nurse to be able to assess. The client's complaint of "bloody diarrhea" needs to be investigated by the nurse, who should observe the amount, color, and characteristics of the stool. 2. Antibiotic therapy is initiated in only the most serious cases of infectious diarrhea; the diarrhea must be assessed first. A specimen for culture should be obtained, if possible, before beginning medication. 3. A complete blood count will provide an estimate of blood loss, but it is not the first intervention. 4. An antidiarrheal medication would be administered after the specimen collection. TEST-TAKING HINT: All answer options in a priority-setting question may be actions the nurse would take, but the right answer will be the one (1) that should be taken before the others. Collecting a stool sample is assessment, which is the first step in the nursing process.

The client has had a stool that is dark, watery, and shiny in appearance. Which intervention should be the nurse's first action? 1. Check for a fecal impaction. 2. Encourage the client to drink fluids. 3. Check the chart for sodium and potassium levels. 4. Apply a protective barrier cream to the perianal area.

***1. This is a symptom of diarrhea moving around an impaction higher up in the colon. The nurse should assess for an impaction when observing this finding. 2. Encouraging the client to drink fluids should be done, but this is not the first intervention. 3. The sodium level is usually not a problem for clients experiencing diarrhea, but the potassium level may be checked. However, again, this is not the first intervention. 4. A protective cream can be applied to an excoriated perineum, but first the nurse should assess the situation. TEST-TAKING HINT: The first step of the nursing process is assessment, after which a nursing diagnosis and interventions follow. The nurse should assess first.

The nurse is assessing the client in end-stage liver failure who has been diagnosed with portal hypertension. Which intervention should the nurse include in the plan of care? 1. Assess the abdomen for a tympanic wave. 2. Monitor the client's blood pressure. 3. Percuss the liver for size and location. 4. Weigh the client twice each week.

**1. A client who has been diagnosed with portal hypertension should be assessed for a fluid wave to check for ascites. 2. High blood pressure is not the etiology of portal hypertension. 3. In portal hypertension, the liver usually cannot be percussed. 4. Weighing the client should be done daily, not twice each week.

When assessing the oral cavity of an elderly client, which data should the nurse report to the health-care provider? 1. The client's tongue is rough and beefy red. 2. The client's tonsils are at a 1 on a grading scale. 3. The client's mucosa is pink and moist. 4. The client's uvula rises with the mouth open.

**1. A rough, beefy-red tongue may indicate that the client has pernicious anemia and should be evaluated by the health-care provider. 2. A score of 1 on the tonsil grading scale shows that the tonsils are extending to the pharyngopalatine arch, which is normal. 3. Mucosa should be pink and moist; therefore the nurse would not need to notify the healthcare provider. 4. Symmetrical movement of the uvula is normal and should not be reported to the health-care provider.

The client receiving antibiotic therapy complains of white, cheesy plaques in the mouth that bleed when removed. Which action should the nurse implement? 1. Notify the health-care provider to obtain an antifungal medication. 2. Explain that the patches will go away naturally in about two (2) weeks. 3. Instruct the client to rinse the mouth with diluted hydrogen peroxide and water daily. 4. Allow the client to verbalize feelings about having the plaques.

**1. Candidiasis, or thrush, presents as white, cheesy plaques that bleed if rubbed and is a side effect of antibiotic therapy. Candidiasis is treated with antifungal solution, which is swished around the mouth, held for at least one (1) minute, and then swallowed. Candidiasis can be prevented if acidophilus is administered concurrently with antibiotic therapy. 2. White painless patches that disappear in approximately two (2) weeks are leukoplakia, caused by tobacco use, which may be cancerous and should be evaluated by an HCP. 3. A solution of hydrogen peroxide is not recommended to treat candidiasis. 4. The nurse needs to treat the client's mouth, not listen to feelings.

The client diagnosed with a hiatal hernia has been scheduled for a laparoscopic Nissen fundoplication. Which statement indicates that the nurse's teaching has been effective? 1. "I will have three (3) or four (4) small incisions." 2. "I will be able to go home the same day of surgery." 3. "I will not have any pain because this is laparoscopic surgery." 4. "I will be returning to work the day after my surgery."

**1. In a laparoscopic Nissen fundoplication, there are four (4) to five (5) incisions approximately one (1) inch apart that allow for the passage of equipment to visualize the abdominal organs and perform the operation. 2. Many clients come through the day surgery department, but some must remain in the hospital for more than one (1) day, so this may be a false statement. 3. There is no surgery that does not cause pain. 4. This is still surgery and the client should not return to work the next day; the client should wait at least one (1) week before returning to work.

The nurse writes the problem "risk for impaired skin integrity" for a client with a sigmoid colostomy. Which expected outcome would be appropriate for this client? 1. The client will have intact skin around the stoma. 2. The client will be able to change the ostomy bag. 3. The client will express anxiety about the body changes. 4. The client will maintain fluid balance

**1. Intact skin around the stoma is the most appropriate outcome for the problem of "impaired skin integrity." 2. Being able to change the ostomy bag is a goal for a knowledge-deficit problem. 3. Expressing anxiety about the body changes would be a goal for an alteration in body image. 4. Maintaining a balance in fluid would be a goal for a nursing diagnosis of risk for fluid deficit.

The nurse working in a skilled nursing facility is collaborating with the dietician concerning the meals of a client who is immobile. Which foods would be most appropriate for this client? 1. Oatmeal and wheat toast. 2. Cream of wheat and biscuits. 3. Cottage cheese and canned peaches. 4. Tuna on croissant and applesauce.

**1. Oatmeal and wheat toast are high-fiber foods that are recommended for clients who are immobile to help prevent constipation. 2. Cream of wheat and biscuits are low-fiber foods. 3. Cottage cheese and canned peaches are lowfiber foods. 4. Tuna is a good source of protein for the client, but croissants have a high fat content and would be a factor in weight gain if consistently eaten. Applesauce is low in fiber.

The nurse in an outpatient clinic is caring for a client who is 67 inches tall and weighs 100 kg. The client complains of occasional pyrosis that resolves with standing or with taking antacids. What treatment should the nurse expect the HCP to order? 1. Place the client on a weight loss program. 2. Instruct the client to eat three (3) balanced meals. 3. Tell the client to take an antiemetic before each meal. 4. Discuss the importance of decreasing alcohol intake.

**1. Obesity increases the risk of pyrosis (heartburn); therefore losing weight could help decrease the incidents. 2. Eating small, frequent meals along with decreased intake of spicy foods have been linked to the prevention of heartburn (pyrosis). 3. Antiemetics decrease nausea, which is not a problem with heartburn. Antacids neutralize the acid of the stomach and are used to treat heartburn. 4. Drinking alcoholic beverages increases heartburn and should be avoided, not decreased.

The elderly client has been diagnosed with acute gastritis. Which client problem would be priority for this client? 1. Fluid volume deficit. 2. Altered nutrition: less than body requirements. 3. Impaired tissue perfusion. 4. Alteration in comfort.

**1. Pediatric and geriatric clients are the most at risk for fluid volume and electrolyte imbalances, and the nurse should always be alert to this possible complication. 2. Altered nutrition may be appropriate, depending on how long the client has been unable to eat, but it would not be priority over fluid volume deficit. 3. Impaired tissue perfusion may be appropriate if the mucosal lining of the stomach is unable to heal, but it is not priority over fluid volume deficit. 4. Alteration in comfort may be appropriate, but is not priority over fluid volume deficit.

Which expected outcome would be appropriate for the client diagnosed with aphthous stomatitis? 1. The client will be able to cope with perceived stress. 2. The client will consume a balanced diet. 3. The client will deny any difficulty swallowing. 4. The client will take antacids as prescribed.

**1. The cause of canker sores, aphthous stomatitis, is unknown. The small ulcerations of the soft oral tissue are linked to stress, trauma, allergies, viral infections, and metabolic disorders. Therefore, being able to cope with stress would be a desired outcome. 2. The client with recurrent erythematous macule cankers will not have malnutrition; therefore a balanced diet would not be applicable to this client. 3. The client with cankers should not have difficulty swallowing. 4. Antacids are not a treatment for canker sores.

Which intervention should the nurse implement specifically for the client in end-stage liver failure who is experiencing hepatic encephalopathy? 1. Assess the client's neurological status. 2. Prepare to administer a loop diuretic. 3. Check the client's stool for bleeding. 4. Assess the abdominal fluid wave.

**1. The increased serum ammonia level associated with liver failure causes the hepatic encephalopathy, which, in turn, leads to neurological deficit. 2. Administering a loop diuretic would be appropriate for ascites and portal hypertension. 3. Checking the stool for bleeding would be appropriate for esophageal varices and decreased vitamin K. 4. Assessing the abdominal fluid wave would be appropriate for ascites and portal hypertension.

The nurse is caring for the client that is one (1)-day post-upper gastrointestinal series (UGI). Which assessment data warrant immediate intervention? 1. No bowel movement. 2. Oxygen saturation 96%. 3. Vital signs within normal baseline. 4. Intact gag reflex.

**1. The nurse should monitor the client for the first bowel movement to document elimination of barium that should be eliminated within two (2) days. If the client does not have a bowel movement, a laxative may be needed to help the client to eliminate the barium before it becomes too hard to pass. 2. An oxygen saturation of 96% is acceptable and does not require intervention. 3. Vital signs should be monitored to recognize and treat complications before the client is in danger. Baseline is a desired outcome. 4. The client's throat is not anesthetized for this procedure, so the gag reflex is not pertinent information in this procedure.

The nurse is teaching the client diagnosed with colon cancer who is scheduled for a colostomy the next day. Which behavior indicates the best method of applying adult teaching principles? 1. The nurse repeats the information as indicated by the client's questions. 2. The nurse teaches in one session all the information that the client needs. 3. The nurse uses a video so that the client can hear the medical terms. 4. The nurse waits until the client asks questions about the surgery.

**1. The nurse should realize the client is anxious about the diagnosis of cancer and the impending surgery. Therefore the nurse should be prepared to repeat information as necessary. The teaching principle that the nurse needs to consider is that anxiety decreases learning. 2. Small manageable sessions increase learning, especially when the client is anxious. 3. Videos are not the best teaching tool for adults. Short videos are useful for children. 4. The nurse should assess the client's readiness and willingness to learn and not wait until the client asks questions about the surgery.

The nurse is speaking to a support group for clients diagnosed with Crohn's disease. Which information would be most important for the nurse to discuss with the clients? 1. Discuss coping skills that assist with adaptation to lifestyle modifications. 2. Teach about drug administration, dosages, and scheduled times. 3. Teach dietary changes necessary to control symptoms. 4. Explain the care of the colostomy and necessary equipment.

**1. The objectives for support groups are to help members cope with chronic diseases and help manage symptom control. 2. Drug administration, dosage, and scheduled times should be discussed in the hospital prior to discharge or in the health-care provider's office; therefore, it is not a priority at the group support meeting. 3. Dietary changes should be taught at the time the disease is diagnosed, but it is not a priority at the group support meeting. 4. Colostomy may be the surgical option for clients who do not respond to medical treatment, but other nonsurgical treatments would be topics

The nurse is preparing the postoperative nursing care plan for the client recovering from a hemorrhoidectomy. Which intervention should the nurse implement? 1. Establish a rapport with the client to decrease embarrassment of assessing site. 2. Encourage the client to lie in the lithotomy position twice a day. 3. Milk the tube inserted during surgery to allow the passage of flatus. 4. Digitally dilate the rectal sphincter to express old blood.

**1. The site of the surgery can cause embarrassment when the nurse assesses the site; therefore, the nurse should establish a positive relationship. 2. The lithotomy position is with the client's legs in stirrups for procedures such as Pap smears and some surgeries such as transurethral resection of the prostate, not for the client who is postoperative hemorrhoidectomy. 3. A tube is not placed in the client's rectum after this surgery. 4. The rectal sphincter does not need to be digitally dilated.

Which nursing diagnosis would be appropriate for the nurse to identify for the client with diarrhea? 1. Alteration in skin integrity. 2. Chronic pain perception. 3. Fluid volume excess. 4. Ineffective coping.

**1. When clients have multiple liquid stools, the rectal area can become irritated. The integrity of the skin can be impaired. 2. Pain experienced by this client would be acute, rather than chronic. 3. Fluid volume deficit would be appropriate, rather than fluid volume excess. 4. Ineffective coping is a psychosocial problem and is not appropriate for a client with diarrhea.

The nurse is caring for the following clients on a surgical unit. Which client would the nurse assess first? 1. The client who had an inguinal hernia repair and has not voided in four (4) hours. 2. The client who was admitted with abdominal pain who suddenly has no pain. 3. The client four (4) hours postoperative abdominal surgery with no bowel sounds. 4. The client who is one (1) day postoperative appendectomy who is being discharged.

1. A client who has not voided within four (4) hours after any surgery would not be priority. This is an acceptable occurrence, but if the client hasn't voided for eight (8) hours, then the nurse would assess further. ***2. This could indicate a ruptured appendix, which could lead to peritonitis, a lifethreatening complication; therefore, the nurse should assess this client first. 3. Bowel sounds should return within 24 hours after abdominal surgery. Absent bowel sounds at four (4) hours postoperative would not be of great concern to the nurse. 4. The client being discharged would be stable and not a priority for the nurse. TEST-TAKING HINT: The stem is asking which client the nurse should see first. Therefore, the test taker should look for life-threatening or serious complications or abnormal assessment data for the disease process.

The nurse is caring for the client diagnosed with ascites from hepatic cirrhosis. What information should the nurse report to the health-care provider? 1. A decrease in the client's daily weight of one (1) pound. 2. An increase in urine output after administration of a diuretic. 3. An increase in abdominal girth of two (2) inches. 4. A decrease in the serum direct bilirubin to 0.6 mg/dL.

1. A decrease in weight would indicate a loss in fluid and would not be reported to the healthcare provider. 2. An increase in urine output would indicate that the diuretic was effective. **3. An increase in abdominal girth would indicate that the ascites is increasing, meaning that the client's condition is becoming more serious and should be reported to the health-care provider. 4. The normal direct bilirubin value is 0.1 to 0.4 mg/dL; therefore, a decrease in the value although it is still elevated would not be reported.

The nurse is teaching a class on diverticulosis. Which interventions should the nurse discuss when teaching ways to prevent an acute exacerbation of diverticulitis? Select all that apply. 1. Eat a low-fiber diet. 2. Drink 2500 mL of water daily. 3. Avoid eating foods with seeds. 4. Walk 30 minutes a day. 5. Take an antacid every two (2) hours.

1. A high-fiber diet will prevent constipation, the primary reason for diverticulitis. A low-fiber (residue) diet would be prescribed for acute diverticulitis. **2. Increased fluids will help keep the stool soft and prevent constipation. **3. It is controversial if seeds cause an exacerbation of diverticulosis, but this is an appropriate intervention to teach until proved otherwise. **4. Exercise will help prevent constipation, which causes an exacerbation of diverticulitis. 5. There are no medications used to help prevent an acute exacerbation of diverticulitis. Antacids are used to neutralize hydrochloric acid in the stomach.

The client is complaining of painful swallowing secondary to mouth ulcers. Which statement indicates the nurse's teaching has been effective? 1. "I will brush my teeth with a soft-bristle toothbrush." 2. "I will rinse my mouth with Listerine mouth wash." 3. "I will swish with antifungal solution and then swallow." 4. "I will avoid spicy foods, tobacco, and alcohol."

1. A soft-bristle toothbrush will not affect painful swallowing. 2. An alcohol mouthwash would be irritating to the oral cavity and increase pain. 3. An antifungal medication should be used with candidiasis and would not be an effective treatment for plain mouth ulcers. **4. Substances that are irritating should be avoided during the outbreaks of ulcers in the mouth. Spicy foods, alcohol, and tobacco are common irritants that the client should avoid.

The nurse is caring for the client diagnosed with chronic gastritis. Which symptom(s) would support this diagnosis? 1. Rapid onset of mid-sternal discomfort. 2. Epigastric pain relieved by eating food. 3. Dyspepsia and hematemesis. 4. Nausea and projectile vomiting.

1. Acute gastritis is characterized by sudden epigastric pain or discomfort, not mid-sternal chest pain. **2. Chronic pain in the epigastric area that is relieved by ingesting food is a sign of chronic gastritis. 3. Dyspepsia (heartburn) and hematemesis (vomiting blood) are frequent symptoms of acute gastritis. 4. Projective vomiting is not a sign of chronic gastritis.

The client being admitted from the emergency department is diagnosed with a fecal impaction. Which nursing intervention should be implemented? 1. Administer an antidiarrheal medication, every day and PRN. 2. Perform bowel training every two (2) hours. 3. Administer oil retention enemas. 4. Prepare for an upper gastrointestinal (UGI) series x-ray.

1. An antidiarrheal medication would slow down the peristalsis in the colon, worsening the problem. 2. The client has an immediate need to evacuate the bowel, not bowel training. ***3. Oil retention enemas will help to soften the feces and evacuate the stool. 4. A UGI series would add barium to the already hardened stool in the colon. Barium enemas x-ray the colon; a UGI x-rays the stomach and jejunum. TEST-TAKING HINT: If the test taker understands that fecal impaction is the opposite of diarrhea, then option "1" can be eliminated. Knowledge of anatomy and physiology eliminates "4" because stool is formed in the colon and transported to the anus, part of the lower gastrointestinal tract.

The nurse is caring for the client diagnosed with hepatic encephalopathy. Which sign and symptom would indicate that the disease is progressing? 1. The client has a decrease in serum ammonia level. 2. The client is not able to circle choices on the menu. 3. The client is able to take deep breaths as directed. 4. The client is now able to eat previously restricted food items.

1. An increase in serum ammonia levels is seen in clients that have hepatic encephalopathy and coma. **2. The inability to circle food items on the menu indicates deterioration in the client's cognitive status. 3. Being able to follow commands indicates that the client's neurological status is intact. 4. Consuming foods that provide adequate nutrition indicates the client is getting better and able to follow client teaching.

Which statement made by the client admitted with electrolyte imbalance from frequent cathartic use demonstrates an understanding of the discharge teaching? 1. "In the future I will eat a banana every time I take the medication." 2. "I don't have to have a bowel movement every day." 3. "I should limit the fluids I drink with my meals." 4. "If I feel sluggish, I will eat a lot of cheese and dairy products."

1. Bananas are encouraged for clients with potassium loss from diuretics; a banana is not needed for harsh laxative (cathartic) use. Harsh laxatives should be discouraged because they cause laxative dependence and a narrowing of the colon with long-term use. ***2. It is not necessary to have a bowel movement every day to have normal bowel functioning. 3. Limiting fluids will increase the problem; the client should be encouraged to increase the fluids in the diet. 4. If the client were feeling "sluggish" from not being able to have a bowel movement, these foods would increase constipation because they are low in residue (fiber). TEST-TAKING HINT: The test taker must understand terms such as "cathartic." Limiting fluids is used for clients in renal failure or congestive heart failure, but increasing fluids is recommended for most other conditions.

The client has been experiencing difficulty and straining when expelling feces. Which intervention should be taught to the client? 1. Explain that some blood in the stool will be normal for the client. 2. Instruct the client in manual removal of feces. 3. Encourage the client to use a cathartic laxative on a daily basis. 4. Place the client on a high-residue diet.

1. Blood may indicate a hemorrhoid, but it is not normal to expel blood when having a bowel movement. 2. Nurses manually remove feces; it is not a selfcare activity. 3. Cathartic use on a daily basis creates dependence and a narrowing of the lumen of the colon, creating a much more serious problem. ***4. A high-residue diet provides bulk for the colon to use in removing the waste products of metabolism. Bulk laxatives and fiber from vegetables and bran assist the colon to work more effectively. TEST-TAKING HINT: Blood is not normal in any circumstance. It may be expected but is not "normal" unless inside a vessel.

The dietician and nurse in a long-term care facility are planning the menu for the day. Which foods would be recommended for the immobile clients for whom swallowing is not an issue? 1. Cheeseburger and milk shake. 2. Canned peaches and a sandwich on whole-wheat bread. 3. Mashed potatoes and mechanically ground red meat. 4. Biscuits and gravy with bacon.

1. Cheeseburgers and milk shakes are lowresidue foods that can make constipation worse. ***2. Canned peaches are soft and can be chewed and swallowed easily while providing some fiber, and whole-wheat bread is higher in fiber than white bread. These foods will be helpful for clients whose gastric motility is slowed as a result of lack of exercise or immobility. 3. These foods do not provide the needed fiber. 4. These are refined flour foods or processed meat (fat). These will not help clients to prevent constipation. TEST-TAKING HINT: The test taker must realize that the consequences of immobility include constipation. Then the test taker should try to eliminate processed foods from the choices.

Which complaint would be significant for the nurse to assess in the adolescent male client who uses oral tobacco? 1. The client complains of clear to white sputum. 2. The client has an episodic blister on the upper lip. 3. The client complains of a nonhealing sore in the mouth. 4. The client has bilateral ducts at the second molars.

1. Clear to white sputum would not be significant in the client using oral tobacco. 2. Episodic blisters on the lips are herpes simplex 1 and would not be specific to this client. **3. Presence of any nonhealing sore on the lips or mouth may be oral cancer, the risk for which is increased by using oral tobacco. 4. Bilateral Stensen's ducts visible at the site of the second molars are normal assessment data.

The nurse is caring for a client diagnosed with ulcerative colitis. Which symptom(s) supports this diagnosis? 1. Increased appetite and thirst. 2. Elevated hemoglobin. 3. Multiple bloody, liquid stools. 4. Exacerbations unrelated to stress.

1. Clients suffering from ulcerative colitis have anorexia, not an increased appetite. 2. The hemoglobin and hematocrit would be decreased, not elevated, as a result of blood loss. **3. Clients report as many as 10 to 20 liquid bloody stools in a day. 4. Stressful events have been linked to an increase in symptoms. The nurse needs to assess for perceived stress in the client's life that produces symptoms.

The client has been diagnosed with hemorrhoids. Which statement from the client indicates that further teaching is needed? 1. "I should increase fruits, bran, and fluids in my diet." 2. "I will use warm compresses and take sitz baths daily." 3. "I must take a laxative every night and have a stool daily." 4. "I can use an analgesic ointment or suppository for pain."

1. Clients with hemorrhoids need to eat highfiber diets and increase fluid intake to keep the stools soft and prevent constipation; therefore, the teaching is effective. 2. Warm compresses or sitz baths decrease pain; therefore, the teaching is effective. **3. Laxatives can be harsh to the bowel and are habit-forming, so they should not be taken daily. Stool softeners that soften stool can be taken daily. 4. Analgesic ointments, suppositories, and astringents can be used to decrease pain and decrease edema; therefore, the teaching has been effective.

The nurse is caring for the client scheduled for an abdominal perineal resection for Stage IV colon cancer. When preparing the plan of care during surgery, which client problem should the nurse include in the plan? 1. Fluid volume deficit. 2. Impaired tissue perfusion. 3. Infection of surgical site. 4. Immunosuppression.

1. Fluid deficit would be a potential problem, not an actual problem. The client's fluid balance should be managed by intravenous fluids. Assessment should support the balance. **2. The perfusion of the surgical site is compromised as a result of the surgical incision, especially when a graft is used. 3. Infection would be a potential problem, but not at the time of surgery. 4. After surgery, not during surgery, the client may require chemotherapy, which would cause immunosuppression.

The nurse is caring for the client with active herpes simplex 1 lesions. Which intervention should the nurse implement to prevent the spread of the virus? 1. Wash hands completely only before providing care. 2. Wear clean gloves to prevent transfer of the virus. 3. Scrub the lesions with soap and water twice daily. 4. Apply 1% Lidocaine (hydrocortisone) cream to the lesions.

1. Hand washing before and after care is a minimum measure to prevent transfer of the virus; it is a basic standard precaution. **2. Clean gloves should be worn when providing care to prevent the transfer of the herpes simplex 1 virus. 3. Scrubbing the lesions would transfer the virus along the nerves and spread the disease process. 4. Herpes simplex is treated with acyclovir (Zovirax), an antiviral medication; it is not treated with Lidocaine cream.

Which intervention should the nurse include when discussing ways to prevent food poisoning? 1. Wash hands for ten (10) seconds after handling raw meat. 2. Clean all cutting boards between meats and fruits. 3. Maintain food temperatures at 140F during extended servings. 4. Explain that fruits do not require washing prior to eating or preparing.

1. Hand washing for ten (10) seconds is not long enough to remove any bacteria. Hands should be washed for at least 30 seconds before handling food or eating. 2. Cutting surfaces used for meats should be different from those used for fruits and vegetables to prevent contamination. **3. Foods that are being served for an extended time should be kept at 140F. 4. All fruits and vegetables should be washed before eating or preparing.

The post-anesthesia care nurse is caring for a client who has had abdominal surgery. The client is complaining of nausea. Which intervention should the nurse implement first? 1. Medicate the client with a narcotic analgesic IVP. 2. Assess the nasogastric tube for patency. 3. Check the temperature for elevation. 4. Hyperextend the neck to prevent stridor.

1. Medicating the client with an analgesic could increase the client's nausea unless the nausea is caused by pain. The nurse should assess the etiology to determine the interventions. ***2. A client who has had abdominal surgery usually has a nasogastric tube (NGT) in place. If the NGT is not patent, this will cause nausea. Irrigating the NGT may relieve nausea. 3. Checking the temperature will not treat the nausea. 4. Hyperextending the neck will assist the client to breathe but will not treat nausea. TEST-TAKING HINT: Assessment is the first step in the nursing process. Checking the NGT for patency and taking the temperature are the only assessment activities. Temperature does not correlate with nausea. Medication may be administered but it would be an antiemetic, not a narcotic analgesic.

The nurse is assessing a client complaining of abdominal pain. Which data would support the diagnosis of a bowel obstruction? 1. Steady, aching pain in one specific area. 2. Sharp back pain radiating to the flank. 3. Sharp pain that increases with deep breaths. 4. Intermittent colicky pain near the umbilicus.

1. Steady, aching pain is associated with a peritoneal inflammation, which may be secondary to a ruptured spleen or perforated ulcer or other abdominal organ. 2. Sharp pain in the back and flank indicates kidney involvement. 3. Sharp pain that increases with deep breaths indicates muscular involvement. **4. Intermittent and colicky pain located near the umbilicus is indicative of a small bowel obstruction; lumbar pain is indicative of colon involvement.

The male client with rule out colon cancer is two (2) hours post-sigmoidoscopy procedure. Which intervention would warrant immediate intervention by the nurse? 1. The client has hyperactive bowel sounds. 2. The client is eating a hamburger that his family brought. 3. The client is sleepy and wants to sleep. 4. The client's BP is 96/60 and apical pulse is 108.

1. The client has been NPO and had laxatives; therefore, hyperactive bowel sounds would not warrant immediate intervention. 2. The client is able to eat after the procedure, so this would not warrant immediate intervention. 3. The client received sedation during the procedure. That, plus being up all the night before having bowel movements, may have made the client exhausted and sleepy. **4. These are signs/symptoms of hypovolemic shock that require immediate intervention by the nurse.

The client is one (1) hour postoperative laparoscopic cholecystectomy. Which intervention should the nurse implement? 1. Assess the client's abdominal dressing for bleeding. 2. Monitor the client's T-tube output every one (1) hour. 3. Discuss discharge teaching with the significant other. 4. Check the client's upper right quadrant stoma site.

1. The client has three (3) to four (4) band-aid incisions in the upper quadrant, not an abdominal dressing. 2. The client will not have a T-tube with a laparoscopic cholecystectomy. **3. A laparoscopic cholecystectomy is done in day surgery. The nurse must make sure the significant others taking care of the client are knowledgeable of postoperative care. 4. The client does not have a stoma.

The client is placed on percutaneous gastrostomy (PEG) tube feedings. Which occurrence would warrant immediate intervention by the nurse? 1. The client tolerates the feedings being infused at 50 mL/hour. 2. The client pulls the nasogastric feeding tube out. 3. The client complains of being thirsty. 4. The client has green, watery stool.

1. The client is tolerating the feeding change so there is no need for an immediate action. 2. The client has a PEG tube; therefore, the nasogastric feeding tube can be removed. 3. This should be addressed; the client may require some ice chips in the mouth or some oral care, but this must wait until an assessment of the client's ability to swallow has been completed. ***4. This client needs to be cleaned immediately; the abdomen must be assessed; and a determination must be made regarding the type of feeding and the additives and medications being administered and skin damage occurring. This client is priority. TEST-TAKING HINT: The test taker must identify assessment data that indicate a complication secondary to the disease process when the stem asks which warrants immediate intervention.

Which information should the nurse teach the client post-barium enema procedure? 1. The client should not eat or drink anything for four (4) hours. 2. The client should remain on bed rest until the sedative wears off. 3. The client should take a mild laxative to help expel the barium. 4. The client will have normal elimination color and pattern immediately.

1. The client may resume the regular diet. 2. The client will not be sedated for this procedure; therefore the client does not need to be on bed rest. **3. The nurse needs to teach the client to take a mild laxative to help evacuate the barium and return to the client's normal bowel routine. Failure to pass the barium could cause constipation when the barium hardens. 4. The client can expect to pass white- or lightcolored stools until the barium has completely been evacuated.

The client is diagnosed with peritonitis. Which assessment data indicate the client's condition is improving? 1. The client is using more pain medication on a daily basis. 2. The client's nasogastric tube is draining coffee-ground material. 3. The client has a decrease in temperature and a soft abdomen. 4. The client has had two (2) soft, formed bowel movements.

1. The client needing more pain medication indicates the client's condition is getting worse. 2. Coffee-ground material indicates old blood. ***3. Because the signs of peritonitis are elevated temperature and rigid abdomen, a reversal of these signs would indicate the client is getting better. 4. Two soft-formed bowel movements are good, but this does not have anything to do with peritonitis. TEST-TAKING HINT: The "itis" of peritonitis means inflammation. Peritonitis is inflammation of the peritoneum, or abdominal wall, caused by an infection. An infection is usually associated with an elevated temperature, so a decrease in temperature would be a sign that the client is improving. A soft abdomen is also a good sign. Knowing that the word "peritoneum" means "abdominal wall" could help the test taker choose option "3," which is the only option that contains the word abdomen.

The client has been diagnosed with esophageal diverticula. Which lifestyle modification should be taught by the nurse? 1. Raise the foot of the bed to 45 degrees to increase peristalsis. 2. Eat the evening meal at least two (2) hours prior to bed. 3. Eat a low-fat, low-cholesterol, high-fiber diet. 4. Wear an abdominal binder to strengthen the abdominal muscles.

1. The client needs to elevate the head, not the foot, of the bed to prevent the reflux of stomach contents. **2. The evening meal should be eaten at least two (2) hours prior to retiring. Small, frequent meals and semi-soft foods ease the passage of food, which can decrease signs and symptoms of the disease process. 3. This diet is recommended for a client with coronary artery disease, not for esophageal diverticula. 4. Restrictive clothing should be avoided, and abdominal binders do not strengthen muscles and would not benefit this client.

Which instruction should be discussed with the client diagnosed with gastroesophageal reflux disease (GERD)? 1. Eat a low-carbohydrate, low-sodium diet. 2. Lie down for 30 minutes after eating. 3. Do not eat spicy foods or acidic foods. 4. Drink two (2) glasses of water before bedtime.

1. The client should eat a low-fat, high-fiber diet. 2. The client should not lie down for at least two (2) hours after each meal to prevent gastric reflux. **3. The client should avoid irritants, such as spicy foods or acidic foods, as well as alcohol, caffeine, and tobacco, because they increase gastric secretions. 4. The client should avoid food or drink two (2) hours before bedtime or lying down after eating.

The client has developed a paralytic ileus after abdominal surgery. Which intervention should the nurse include in the plan of care? 1. Administer a laxative of choice. 2. Encourage client to increase oral fluids. 3. Encourage the client to take deep breaths. 4. Maintain a patent nasogastric tube.

1. The client would be NPO; therefore no medication would be administered. 2. The client would be NPO; therefore no food or fluids would be administered. 3. Deep breathing will help prevent pulmonary complications, but it will not address the client's paralytic ileus. ***4. A paralytic ileus is the absence of peristalsis; therefore the bowel will be unable to process any oral intake. A nasogastric tube is inserted to decompress the bowel until there is surgical intervention or bowel sounds return spontaneously. TEST-TAKING HINT: If the test taker realizes that the stem of the question says that the part of the gastrointestinal system, the ileus, is paralyzed, the test taker should know that allowing the client to take anything by mouth would be an inappropriate action, so options "1" and "2" could be eliminated. Deep breathing addresses the respiratory system, not the gastrointestinal system, so "3" could also be eliminated.

The female client is diagnosed with ulcerative colitis. Which sign/symptom would warrant immediate intervention by the nurse? 1. The client has 20 bloody stools a day. 2. The client's oral temperature is 99.8F. 3. The client's abdomen is hard and rigid. 4. The client complains of urinating when she coughs.

1. The colon is ulcerated and unable to absorb water, so 10 to 20 bloody diarrhea stools is the most common symptom of ulcerative colitis and would not warrant immediate intervention. 2. This is not an elevated temperature and would not warrant immediate intervention by the nurse. **3. A hard, rigid abdomen indicates peritonitis, which is a complication of ulcerative colitis and warrants immediate intervention. 4. Stress incontinence is not a symptom of colitis and would not warrant immediate intervention.

nurse identifies the client problem "alteration in gastrointestinal system" for the elderly client. Which statement reflects the most appropriate rationale for this diagnosis? 1. Elderly clients have a better mechanical handling of food with dentures. 2. Elderly clients have an increase in digestive enzymes, which helps with digestion. 3. Elderly clients have an increased need for laxatives because of a decrease in bile. 4. Elderly clients have an increase in bacteria in the GI system, resulting in diarrhea.

1. The elderly client does not have a better way of breaking down food with dentures. 2. The secretion of digestive enzymes and bile is decreased in the elderly, and this results in an alteration in nutrition and elimination. 3. Bile does not affect motility of the intestines. The need for laxatives is caused by client misunderstanding about the frequency of bowel movements and to decreased fluid and fiber intake. **4. When the motility of the gastrointestinal tract decreases, bacteria remain in the gut longer and multiply, which results in diarrhea.

The nurse is working in an emergency department of a community hospital. During the past 2 hours, 15 clients have been diagnosed with Salmonella food poisoning. Which information should the nurse discuss with clients? 1. Explain that the incubation period is 48 to 72 hours. 2. Explain that the source of this poisoning is contaminated water. 3. Explain that one (1) source of potential contamination is eggs. 4. Explain that the bacterial contaminant is from canned foods.

1. The incubation period for Salmonella food poisoning is 8 to 48 hours. 2. Salmonellae bacteria are not transmitted to humans via water. **3. Eggs, poultry, pet turtles, and chickens are sources of the Salmonellae bacteria, which cause food poisoning. 4. Clostridium botulinum is transmitted via improperly canned food.

The 84-year-old client comes to the clinic complaining of right lower abdominal pain. Which question would be most appropriate for the nurse to ask the client? 1. "When was your last bowel movement?" 2. "Did you have a high-fat meal last night?" 3. "How long have you had this pain?" 4. "Have you been experiencing any gas?"

1. The last bowel movement would not help identify the cause of the client's right lower abdominal pain. This might be appropriate for a client with left lower abdominal pain. 2. Information about a high-fat meal would be asked if the nurse suspected the client had a gallbladder problem. ***3. Elderly clients usually display a high tolerance to pain and frequently may have a ruptured appendix with minimal pain, therefore the nurse should assess the characteristic and etiology of the pain. 4. The passage of flatus (gas) does not help determine the cause of right lower abdominal pain. TEST-TAKING HINT: The test taker should go back to basics and assess the client.

The nurse is completing the shift assessment on the client recovering from abdominal surgery who has a PCA pump. The client has shallow respirations and refuses to deep breathe. Which intervention should the nurse implement? 1. Insist that the client take deep breaths. 2. Notify the surgeon to request a chest x-ray. 3. Determine the last time the client used the PCA pump. 4. Administer oxygen 2 L/min via nasal cannula.

1. The nurse cannot force the client to do anything; this would be considered assault. 2. There are no data that support the need for a chest x-ray. ***3. Shallow respirations and refusal to deep breathe could be the result of abdominal pain. The nurse should assess the client for pain and determine the last time the PCA pump was used. 4. Based on the information given, the client does not need oxygen. TEST-TAKING HINT: If the test taker has no idea of what the answer is, identify key words in the stem "abdominal surgery" and "PCA" and select an answer option that is related to one (1) of these key words. "Determine" can be substituted for the word "assess," which is the first step of the nursing process.

The nurse, a licensed practical nurse, and an unlicensed nursing assistant are caring for clients on a medical floor. Which nursing task would be most appropriate to assign to the licensed practical nurse? 1. Assist the unlicensed nursing assistant to learn to perform blood glucose checks. 2. Monitor the potassium levels of a client with diarrhea. 3. Administer a bulk laxative to a client diagnosed with constipation. 4. Assess the abdomen of a client who has had complaints of pain.

1. The nurse will be responsible for signing off on the unlicensed nursing assistant as to being competent to perform the blood glucose. The nurse should do this to determine the competency of the assistant. 2. The lab values may require the nurse to interpret and act on the results. The nurse cannot delegate anything that requires professional judgment. ***3. The licensed practical nurse could administer a laxative. 4. The nurse cannot delegate assessment. TEST-TAKING HINT: State boards of nursing have agreed that nurses cannot delegate any activity that requires professional judgment, assessment, teaching, or evaluation.

The nurse is preparing the client for a fiberoptic colonoscopy for colon polyps. Which task can be delegated to the unlicensed nursing assistant? 1. Administer the polyethylene glycol electrolyte lavage solution. 2. Explain to the client why that morning's breakfast is withheld. 3. Start an intravenous site with 0.9% normal saline fluid. 4. Administer a cleansing enema until the return is clear.

1. The polyethylene glycol electrolyte lavage solution is a medication and therefore cannot be delegated. 2. Teaching is the responsibility of the nurse and cannot be delegated. 3. Starting an intravenous site and managing the delivery of the fluid is the responsibility of the nurse and cannot be delegated. **4. The administration of enemas can be delegated to the unlicensed nursing assistant.

When assessing the integumentary system of the client with anorexia nervosa, which finding would support the diagnosis? 1. Preoccupation with calories. 2. Thick body hair. 3. Sore tongue. 4. Dry, brittle hair.

1. The preoccupation with food, calories, and preparing meals are psychosocial behaviors that suggest the client has an eating disorder. 2. Clients who have anorexia nervosa have thin, fine body hair. 3. Iron deficiency anemia causes clients to experience a sore tongue. **4. Thin, brittle hair occurs in clients with anorexia.

The nurse is caring for the client one (1) day postoperative sigmoid colostomy operation. Which independent nursing intervention should the nurse implement? 1. Change the infusion rate of the intravenous fluid. 2. Encourage the client to discuss his or her feelings. 3. Administer opioid narcotic medications for pain management. 4. Assist the client out of bed to sit in the chair twice daily.

1. The rate of the intravenous fluid is a collaborative nursing intervention because it requires an order from the health-care provider. **2. Encouraging the client to verbalize feelings about body changes assists the client to accept these changes. 3. Medication administration is a collaborative intervention because it requires an order by the health-care provider. 4. Activity level immediately postoperative requires an order by the health-care provider

The nurse is planning the care of a client diagnosed with infectious diarrhea. Which independent problem should be included in the plan of care? 1. Risk for hypovolemic shock. 2. Bacteremia. 3. Fluid volume deficit. 4. Increased knowledge of transmission.

1. The risk for hypovolemic shock would require that the HCP be informed and that there be an order for treatment. It is a collaborative problem. 2. Bacteremia is a bacterial infection in the circulatory system. This is a collaborative problem. ***3. The treatment of a fluid volume deficit is an independent nursing problem; the nurse can assess and intervene with oral fluids. 4. The client might have a decreased knowledge of transmission. Increase in knowledge is not a problem. TEST-TAKING HINT: The question is asking for an independent nursing problem; only one (1) problem appears in any text as a nursing problem/diagnosis. Knowledge problems are always written as knowledge deficit.

The nurse identifies the client problem of "fluid volume deficit" for a client diagnosed with gastritis. Which intervention should be included in the plan of care? 1. Obtain permission for a blood transfusion. 2. Prepare the client for total parenteral nutrition. 3. Monitor the client's lung sounds every shift. 4. Assess the client's intravenous site.

1. There are no data that suggest the client needs a blood transfusion. 2. TPN is not a treatment for a client with fluid volume deficit. TPN provides calories for nutritional deficits, not fluid deficits. 3. If the client's problem was fluid volume excess, assessing lung sounds would be appropriate. **4. This would be the most appropriate intervention to implement because fluid infusion is the treatment of choice for this problem. The effectiveness of the client's treatment would be altered if the intravenous site becomes infected or infiltrated.

The nurse is caring for the client recovering from intestinal surgery. Which assessment finding would require immediate intervention? 1. Presence of thin pink drainage in the Jackson Pratt. 2. Guarding when the nurse touches the abdomen. 3. Tenderness around the surgical site during palpation. 4. Complaints of chills and feeling feverish.

1. Thin pink drainage would be expected in the Jackson Pratt (JP) bulb. 2. Guarding would be a normal occurrence when touching a tender area on the abdomen and would not require immediate intervention. 3. Tenderness around the surgical site would be a normal finding and would not require intervention. **4. Complaints of chills, sudden onset of fever, tachycardia, nausea, and hiccups are symptoms of peritonitis, which is a life-threatening complication.

The charge nurse has completed report. Which client should be seen first? 1. The client diagnosed with Crohn's disease who had two (2) semi-formed stools on the previous shift. 2. The elderly client admitted from another facility who is complaining of constipation. 3. The client diagnosed with AIDS who had a 200-mL diarrhea stool and has elastic skin tissue turgor. 4. The client diagnosed with hemorrhoids who had some spotting of bright red blood on the toilet tissue.

1. This client is improving; semi-formed stools are better than diarrhea. ***2. This client has just arrived so the nurse does not know if the complaint is valid and needs intervention unless this client is seen and assessed. The elderly have difficulty with constipation as a result of decreased gastric motility, medications, poor diet, and immobility. 3. The client has diarrhea, but only 200 mL, and has elastic tissue turgor that lets the nurse know the client is not dehydrated. 4. This is not normal, but it is expected for a client with hemorrhoids. TEST-TAKING HINT: The test taker should notice descriptive words such as "elderly," which should alert the test taker that the age range has an implication in answering the question. Answer options "3" and "4" are expected for the disease processes.

Which priority teaching information should the nurse discuss with the client to help prevent contracting hepatitis B? 1. Explain the importance of good hand washing. 2. Tell the client to take the hepatitis B vaccine in three (3) doses. 3. Tell the client not to ingest unsanitary food or water. 4. Discuss how to implement standard precautions.

1. This would be appropriate for prevention of hepatitis A. **2. The hepatitis B vaccine will prevent the client from contracting this disease. 3. This would be appropriate for prevention of hepatitis A. 4. The nurse uses standard precautions, not the client.

Which task can the nurse delegate to the unlicensed nursing assistant to improve the desire to eat in a client diagnosed with anorexia? 1. Administer an antiemetic 30 minutes before the meal. 2. Provide mouth care with lemon glycerin swabs prior to the meal. 3. Create a social atmosphere by interacting with the client. 4. Encourage the client's parents to sit with the client during meals.

1. Unlicensed nursing assistants cannot administer medications, and this would not be appropriate for a client with anorexia. 2. Mouth care should be provided before and after meals, but not with alcohol mouth wash and lemon glycerin swabs, which decrease the appetite. **3. The NA assisting the client with meals needs to increase interaction to improve the client's appetite and make it an enjoyable occasion. 4. Often the parents are the cause of the client's stress and anxiety, which may have led to the client's anorexia; therefore, the parents should not be asked to stay with the client.

The client is diagnosed with an acute exacerbation of IBD. Which priority intervention should the nurse implement first? 1. Weigh the client daily and document it in the client's chart. 2. Teach coping strategies such as dietary modifications. 3. Record the frequency, amount, and color of stools. 4. Monitor the client's oral fluid intake every shift.

1. Weighing the client daily will help identify if the client is experiencing malnutrition, but it is not the priority intervention during an acute exacerbation. 2. Coping strategies help develop healthy ways to deal with this chronic disease that has remissions and exacerbations, but it is not the priority intervention. ****3. The severity of the diarrhea helps determine the need for fluid replacement. The liquid stool should be measured as part of the total output. 4. The client will be NPO when there is an acute exacerbation of IBD to allow the bowel to rest. TEST-TAKING HINT: The test taker can apply Maslow's Hierarchy of Needs and select the option that is the intervention addressing a physiological need.

The nurse is preparing to hang a new bag of total parental nutrition on a client that has had an abdominal perineal resection. The bag has 1500 mL of 50% dextrose, 10 mL of trace elements, 20 mL of multivitamins, 20 mL of potassium chloride, and 500 mL of lipids. The bag is to infuse over the next 24 hours. At what rate should the nurse set the pump? _______

85 mL/hours. First determine the total amount to be infused over 24 hours. 1500 500 20 20 2040 mL over 24 hours. Then, determine the rate per hour. 2040 ÷ 24 85 mL/hour TEST-TAKING HINT: Check and recheck calculations. Division should be carried out to the second or third decimal place before rounding.

The nurse is preparing to administer a 250-mL intravenous antibiotic to the client. The medication must infuse in one (1) hour. An intravenous pump is not available and the nurse must administer the medication via gravity with IV tubing 10 gtts/min. At what rate should the nurse infuse the medication?_______

42 gtts per minute. The nurse must use the formula: amount to be infused drops per minute ÷ minutes for infusion 41.8 gtts per minute TEST-TAKING HINT: The test taker must know how to calculate dosage and calculation questions. Remember to use the drop-down calculator if needed; the test taker can also ask for an erase slate during state board examinations.

The nurse is caring for an adult client diagnosed with gastroesophageal reflux disease (GERD). Which condition is the most common comorbid disease associated with GERD? 1. Adult-onset asthma. 2. Pancreatitis. 3. Peptic ulcer disease. 4. Increased gastric emptying.

*****1. Of adult-onset asthma cases, 80%-90% are caused by gastroesophageal reflux disease (GERD). 2. Pancreatitis is not related to GERD. 3. Peptic ulcer disease is related to H. pylori bacterial infections and can lead to increased levels of gastric acid, but it is not related to reflux. 4. GERD is not related to increased gastric emptying. Increased gastric emptying would be a benefit to a client with decreased functioning of the lower esophageal sphincter. TEST-TAKING HINT: Adjectives such as "increased" should always be examined to determine how to interpret the answer option.

The client two (2) hours postoperative laparoscopic cholecystectomy is complaining of severe pain in the right shoulder. Which nursing intervention should the nurse implement? 1. Apply a heating pad to the abdomen for 15 to 20 minutes. 2. Administer morphine sulfate intravenously after diluting with saline. 3. Contact the surgeon for an order to x-ray the right shoulder. 4. Apply a sling to the right arm that was injured in surgery.

****1. A heating pad should be applied for 15 to 20 minutes to assist the migration of the CO2 used to insufflate the abdomen. 2. Morphine sulfate would not affect the etiology of the pain. 3. The surgeon would not order an x-ray for this condition. 4. There is no indication that an injury occurred during surgery. A sling would not benefit the migration of the CO2. TEST-TAKING HINT: The test taker must understand laparoscopic surgery to be able to answer this question. Option "4" could be eliminated because of the "injured during surgery" phrase that is making an assumption.

The 85-year-old male client diagnosed with cancer of the colon asks the nurse, "Why did I get this cancer?" Which statement is the nurse's best response? 1. Cancer of the colon is associated with a lack of fiber in the diet. 2. Cancer of the colon has a greater incidence among those younger than age 50 years. 3. Cancer of the colon has no known risk factors. 4. Cancer of the colon is rare among male clients.

****1. A long history of low-fiber, high-fat, highprotein diets results in a prolonged transit time. This allows the carcinogenic agents in the waste products to have a greater exposure to the lumen of the colon. 2. The older the client, the greater the risk of developing cancer of the colon. 3. Risk factors for cancer of the colon include increasing age; family history of colon cancer or polyps; history of IBD; genital or breast cancer; and eating a high-fat, high-protein, low-fiber diet. 4. Males have a slightly higher incidence of colon cancers than do females. TEST-TAKING HINT: The test taker should realize that cancers in general have an increasing incidence with age. Cancer etiologies are not an exact science, but most cancers have some risk factor, if only advancing age

When assessing the client recovering from an open cholecystectomy, which signs and symptoms should the nurse report to the health-care provider? Select all that apply. 1. Clay-colored stools. 2. Yellow-tinted sclera. 3. Dark yellow urine. 4. Feverish chills. 5. Abdominal pain.

****1. Clay-colored stools are caused by recurring stricture of the common bile duct, which is a sign of post-cholecystectomy syndrome. ****2. Yellow-tinted sclera and skin indicate residual effects of stricture of the common bile duct, which is a sign of post-cholecystectomy syndrome. ****3. Dark yellow urine indicates a residual effect of a stricture of the common bile duct, which is a sign of post-cholecystectomy syndrome. ****4. Fever and chills indicate residual or recurring calculi, inflammation, or stricture of common bile duct, which is a sign of postcholecystectomy syndrome. ****5. Abdominal pain indicates a residual effect of a stricture of common bile duct, inflammation, or calculi, which is a sign of postcholecystectomy syndrome. TEST-TAKING HINT: The test taker must use knowledge of anatomy to answer this question. All answer options have something to do with the abdominal area, and the common bile duct is anatomically near the hepatic

The nurse is planning the care of a client who has had an abdominal perineal resection for cancer of the colon. Which interventions should the nurse implement? Select all that apply. 1. Provide meticulous skin care to stoma. 2. Assess the flank incision. 3. Maintain the indwelling catheter. 4. Irrigate the J-P drains every shift. 5. Position the client semi-recumbent.

****1. Colostomy stomas are portions of the large intestines pulled through the abdominal wall through which feces exits the body. Feces can be irritating to the abdominal skin, so careful and thorough skin care is needed. 2. There are midline and perineal incisions, not flank incisions. ****3. Because of the perineal wound, the client will have an indwelling catheter to keep urine out of the incision. 4. Jackson Pratt drains are emptied every shift, but they are not irrigated. ****5. The client should not sit upright because this would cause pressure on the perineum. TEST-TAKING HINT: The test taker could eliminate "2" because flank and abdominal/perineal are not in the same areas. This is an alternative question requiring the test taker to choose more than one (1) option.

Which medication should the nurse question before administering to the client with peptic ulcer disease? 1. E-mycin, an antibiotic. 2. Prilosec, a proton pump inhibitor. 3. Flagyl, an antimicrobial agent. 4. Tylenol, a nonnarcotic analgesic.

****1. E-mycin is irritating to stomach, and its use in a client with peptic ulcer disease should be questioned. 2. Prilosec, a proton pump inhibitor, decreases gastric acid production and its use should not be questioned by the nurse. 3. Flagyl, an antimicrobial, is given to treat peptic ulcer disease secondary to H. pylori bacteria. 4. Tylenol can be safely administered to a client with peptic ulcer disease. TEST-TAKING HINT: The test taker needs to understand how medications work, adverse effects of medications, when to question the safety of giving a specific medication, and how to administer the medication safely. By learning classifications, the test taker should be able to make a knowledgeable selection in most cases.

Which nursing problem is priority for the 76-year-old client diagnosed with gastroenteritis from staphylococcal food poisoning? 1. Fluid volume deficit. 2. Nausea. 3. Risk for aspiration. 4. Impaired urinary elimination.

****1. Fluid volume deficit secondary to diarrhea is the priority because of the potential for metabolic acidosis and hypokalemia, which are both life threatening, especially in the elderly. 2. Nausea may occur, but it is not priority. However, excessive vomiting could lead to potential complications. 3. Risk for aspiration could result from vomiting; however, vomiting does not usually occur in food poisoning, but it may be secondary to botulism. 4. Impaired urinary elimination is not a priority. The client has diarrhea, not urine output problems. TEST-TAKING HINT: Always notice the client's age because it is usually a significant clue as to what the correct answer will be. Prioritizing questions may have more than one (1) potential appropriate nursing problem but only one (1) has priority. Remember Maslow's Hierarchy of Needs

The client with acute diverticulitis has a nasogastric tube draining green liquid bile. Which action should the nurse implement? 1. Document the findings as normal. 2. Assess the client's bowel sounds. 3. Determine the client's last bowel movement. 4. Insert the N/G tube at least 2 more inches.

****1. Green bile contains hydrochloric acid and should be draining from the N/G tube; therefore the nurse should take no action and should document the findings. 2. There is no reason for the nurse to take further action because this is normal. 3. The client's last bowel movement would not affect the N/G drainage. 4. Bile draining from the N/G tube indicates that the tube is in the stomach and there is no need to advance the tube further. TEST-TAKING HINT: The test taker must know what drainage is normal for tubes inserted into the body. Any type of blood or coffee-ground drainage would be abnormal and require intervention by the nurse.

The client diagnosed with liver failure is experiencing pruritus secondary to severe jaundice. Which action by the unlicensed assistant warrants intervention by the primary nurse? 1. Assisting the client to take a hot soapy shower. 2. Applying an emollient to the client's legs and back. 3. Putting mittens on both hands of the client. 4. Patting the client's skin dry with a clean towel.

****1. Hot water increases pruritus, and soap will cause dry skin, which increases pruritus; therefore, the nurse should discuss this with the assistant. 2. This will help prevent dry skin, which will help decrease pruritus; therefore this would not require any intervention by the primary nurse. 3. Mittens will help prevent the client from scratching the skin and causing skin breakdown. 4. The skin should be patted dry, not rubbed, because rubbing the skin will cause increased irritation. TEST-TAKING HINT: A concept that is accepted for most clients during A.M. care is not to use hot water because it causes dilatation of vessels, which may cause orthostatic hypotension. This is not the rationale for not using hot water with a client who has pruritus, but sometimes the test taker can apply broad concepts when answering questions.

The nurse is performing an admission assessment on a client diagnosed with gastroesophageal reflux disease (GERD). Which signs and symptoms would indicate GERD? 1. Pyrosis, water brash, and flatulence. 2. Weight loss, dysarthria, and diarrhea. 3. Decreased abdominal fat, proteinuria, and constipation. 4. Mid-epigastric pain, positive H. pylori test, and melena.

****1. Pyrosis is heartburn, water brash is the feeling of saliva secretion as a result of reflux, and flatulence is gas—all symptoms of GERD. 2. Gastroesophageal reflux disease does not cause weight loss. 3. There is no change in abdominal fat, no proteinuria (the result of a filtration problem in the kidney), and no alteration in bowel elimination for the client diagnosed with GERD. 4. Mid-epigastric pain, a positive H. pylori test, and melena are associated with gastric ulcer disease. TEST-TAKING HINT: Frequently, incorrect answer options will contain the symptoms of a disease of the same organ system.

The client is diagnosed with salmonellosis secondary to eating some slightly cooked hamburger meat. Which clinical manifestations would the nurse expect the client to report? 1. Abdominal cramping, nausea, and vomiting. 2. Neuromuscular paralysis and dysphagia. 3. Gross amounts of explosive bloody diarrhea. 4. Frequent "rice water stool" with no fecal odor.

****1. Symptoms develop 8-48 hours after ingesting the Salmonella bacteria and include diarrhea, abdominal cramping, nausea, and vomiting, along with low-grade fever, chills, and weakness. 2. This occurs with botulism, a severe life-threatening form of food poisoning caused by Clostridium botulinum. 3. This is a clinical manifestation of hemorrhagic colitis caused by Escherichia coli. 4. This gray-cloudy diarrhea that has no fecal odor, blood, or pus is caused by cholera, which is endemic in parts of Asia, the Middle East, and Africa. TEST-TAKING HINT: Often when two (2) answer options have the same clinical manifestation, such as diarrhea and stool, this should make the test taker realize that these two (2) options are either a right or wrong answer so the other two (2) options can be eliminated.

The client diagnosed with IBD is prescribed total parental nutrition (TPN). Which intervention should the nurse implement? 1. Check the client's glucose level. 2. Administer an oral hypoglycemic. 3. Assess the peripheral intravenous site. 4. Monitor the client's oral food intake.

****1. TPN is high in dextrose, which is glucose; therefore the client's blood glucose level must be monitored closely. 2. The client may be on sliding-scale regular insulin coverage for the high glucose level. 3. The TPN must be administered via a subclavian line because of the high glucose level. 4. The client would be NPO to put the bowel at rest, which is the rationale for administering the TPN. TEST-TAKING HINT: The test taker may want to select "3" because it has the word "assess," but remember to note the adjective "peripheral," which makes this option incorrect. Remember the words "check" and "monitor" are words that mean assess.

The client has been seen by the health-care provider and the suspected diagnosis is peptic ulcer disease. Which diagnostic test would confirm this diagnosis? 1. Esophagogastroduodenoscopy (EGD). 2. Magnetic resonance imaging (MRI). 3. Occult blood test. 4. Gastric acid stimulation.

****1. The EGD is an invasive diagnostic test that visualizes the esophagus and stomach to accurately diagnose an ulcer and evaluate the effectiveness of the client's treatment. 2. MRIs show cross-sectional images of tissue or blood flow. 3. An occult blood test shows the presence of blood, but not the source. 4. A gastric acid stimulation test is used to understand the pathophysiology of ulcer disease, but it has limited usefulness. TEST-TAKING HINT: If the test taker has no idea what the correct answer is, knowledge of anatomy can help identify the answer. A peptic ulcer is an ulcer n the stomach, and the word "esophagogastroduodenoscopy" has "gastro," which refers to the stomach. Therefore this would be best option to select as the correct answer.

The nurse caring for a client diagnosed with gastroesophageal reflux disease (GERD) writes the nursing problem of "behavior modification." Which intervention should be included for this problem? 1. Teach the client to sleep with a foam wedge under the head. 2. Encourage the client to decrease the amount of smoking. 3. Instruct the client to take over-the-counter medication for relief of pain. 4. Discuss the need to attend Alcoholics Anonymous to quit drinking.

****1. The client should elevate the head of the bed on blocks or use a foam wedge to use gravity to help keep the gastric acid in the stomach and prevent reflux into the esophagus. Behavior modification is changing one's behavior. 2. The client should be encouraged to quit smoking altogether. Referral to support groups for smoking cessation should be made. 3. The nurse would be prescribing medication; this is not in the nurse's scope of practice. 4. The client should be instructed to discontinue using alcohol, but the stem does not indicate the client is an alcoholic. TEST-TAKING HINT: Clients are not encouraged to decrease smoking. Current research indicates that smoking is damaging to many body systems, including the gastrointestinal system. The nurse must always practice in the scope of practice for which the nurse is licensed. Do not assume anything not in the stem of a question.

The client is diagnosed with ulcerative colitis. When assessing this client, which sign/ symptom would the nurse expect to find? 1. Twenty bloody stools a day. 2. Oral temperature of 102F. 3. Hard, rigid abdomen. 4. Urinary stress incontinence.

****1. The colon is ulcerated and unable to absorb water, resulting in bloody diarrhea. Ten (10) to twenty bloody diarrhea stools is the most common symptom of ulcerative colitis. 2. Although there is an inflammation of the colon, there is usually not an elevated temperature, and if it is elevated, it is a low-grade fever. 3. A hard, rigid abdomen indicates peritonitis, which is a complication of ulcerative colitis but not an expected symptom. 4. Stress incontinence is not a symptom of colitis. TEST-TAKING HINT: If the test taker is not sure of the answer, the test taker should use knowledge of anatomy and physiology to help identify the correct answer. The colon is responsible for absorbing water, and if the colon can't do its job, then water will not be absorbed, causing diarrhea, answer option "3." Colitis is inflammation of the colon; therefore, option "4" referring to the urinary system can be eliminated.

The client with ulcerative colitis is scheduled for an ileostomy. The nurse is aware that the client's stoma will be located in which area of the abdomen? 1. A 2. B 3. C 4. D D C A B

****1. The cure for ulcerative colitis is a total colectomy, which is removing the entire large colon and bringing the terminal end of the ileum up to the abdomen in the right lower quadrant. This is an ileostomy. 2. This is the left-lower quadrant 3. This is the transcending colon. 4. This is the right upper quadrant

The nurse caring for a client one (1) day postoperative sigmoid resection notes a moderate amount of dark reddish brown drainage on the midline abdominal incision. Which intervention should the nurse implement first? 1. Mark the drainage on the dressing with the time and date. 2. Change the dressing immediately using sterile technique. 3. Notify the health-care provider immediately. 4. Reinforce the dressing with a sterile gauze pad.

****1. The nurse should mark the drainage on the dressing to determine if active bleeding is occurring because dark reddish-brown drainage indicates old blood. This allows the nurse to assess what is actually happening. 2. Surgical dressings are initially changed by the surgeon; the nurse should not remove the dressing until the surgeon orders the dressing change to be done by the nurse. 3. The nurse should assess the situation before notifying the HCP. 4. The nurse may need to reinforce the dressing if the dressing becomes too saturated, but this would be after a thorough assessment is completed. TEST-TAKING HINT: The question is asking the test taker to determine which intervention must be implemented first, and assessment is the first step of the nursing process. Options "2," "3," and "4" would not be implemented prior to assessing. Marking the dressing allows the nurse to assess the dressing and determine whether active bleeding is occurring.

The client is diagnosed with Crohn's disease, also known as regional enteritis. Which statement by the client would support this diagnosis? 1. "My pain goes away when I have a bowel movement." 2. "I have bright red blood in my stool all the time." 3. "I have episodes of diarrhea and constipation." 4. "My abdomen is hard and rigid and I have a fever."

****1. The terminal ileum is the most common site for regional enteritis and causes right lower quadrant pain that is relieved by defecation. 2. Stools are liquid or semi-formed and usually do not contain blood. 3. Episodes of diarrhea and constipation may be a sign/symptom of colon cancer, not Crohn's disease. 4. A fever and hard rigid abdomen are signs/ symptoms of peritonitis, a complication of Crohn's disease. TEST-TAKING HINT: The test taker should eliminate option "2" because of the word "all," which is an absolute. There are very few absolutes in the health-care arena.

The client diagnosed with acute diverticulitis is complaining of severe abdominal pain. On assessment, the nurse finds a hard, rigid abdomen and T 102F. Which intervention should the nurse implement? 1. Notify the health-care provider. 2. Prepare to administer a Fleet's enema. 3. Administer an antipyretic suppository. 4. Continue to monitor the client closely.

****1. These are signs of peritonitis, which is life threatening. The health-care provider should be notified immediately. 2. A Fleet's enema will not help a life-threatening complication of diverticulitis. 3. A medication administered to help decrease the client's temperature will not help a life-threatening complication. 4. These are signs/symptoms that indicate a possible life-threatening situation and require immediate intervention. TEST-TAKING HINT: In most instances, the test taker should not select the option that says to notify the HCP immediately, but in some situations, it is the correct answer. The test taker should look at all the other options and determine if the option is information the HCP would require or if it is an independent intervention that will help the client.

Which laboratory value would the nurse expect to find indicating a chronic inflammation in the client with cholecystitis? 1. An elevated white blood cell (WBC) count. 2. A decreased lactate dehydrogenase (LDH) 3. An elevated alkaline phosphatase. 4. A decreased direct bilirubin level.

****1. This value would be elevated in clients with chronic inflammation. 2. This value would indicate liver abnormalities. 3. This value would indicate liver abnormalities. 4. This value would indicate an obstructive process. TEST-TAKING HINT: If the test taker does not know what the values mean, the test taker should look to the disease process. The "itis" means inflammation, and an educated guess would be that WBCs are elevated in inflammatory processes.

Which assessment data should the nurse expect to find for the client who had an upper gastrointestinal (UGI) series? 1. Chalky white stools. 2. Increased heart rate. 3. A firm hard abdomen. 4. Hyperactive bowel sounds.

***1. A UGI requires the client to swallow barium, which passes through the intestines, making the stools a chalky white color. 2. This would be abnormal data and would be cause for further assessment. 3. This would not be expected from the test. 4. This is a serious finding and should be treated. TEST-TAKING HINTS: Answer option "2" could be eliminated because it does not have anything to do with the gastrointestinal system. A firm hard abdomen is very seldom ever expected, so "3" could be eliminated.

The nurse is teaching a class on diverticulosis. Which interventions should the nurse discuss when teaching ways to prevent an acute exacerbation of diverticulosis? Select all that apply. 1. Eat a high-fiber diet. 2. Increase fluid intake. 3. Elevate the HOB after eating. 4. Walk 30 minutes a day. 5. Take an antacid every two (2) hours.

***1. A high-fiber diet will help to prevent constipation, which is the primary reason for diverticulitis. 250 mL 10 gtts 2500 60 minutes ****2. Increased fluids will help keep the stool soft and prevent constipation. 3. This will not do anything to help prevent diverticulitis. 4. Exercise will help prevent constipation. 5. There are no medications used to help prevent an acute exacerbation of diverticulitis. Antacids are used to neutralize hydrochloric acid in the stomach. TEST-TAKING HINT: This is an alternate-type question where the test taker must select all answer options that apply. To prevent diverticulitis, the nurse must discuss interventions that will prevent constipation. To identify the correct answers, the test taker should think about what part of the GI system is affected. Knowing that diverticulosis occurs in the sigmoid colon would help eliminate "3" and "5" because these would be secondary to stomach disorders.

When assessing the client with the diagnosis of peptic ulcer disease, which physical examination should the nurse implement first? 1. Auscultate the client's bowel sounds in all four quadrants. 2. Palpate the abdominal area for tenderness. 3. Percuss the abdominal borders to identify organs. 4. Assess the tender area progressing to nontender.

***1. Auscultation should be used prior to palpation or percussion when assessing the abdomen. If the nurse manipulates the abdomen, the bowel sounds can be altered and give false information. 2. Palpation gives appropriate information that the nurse needs to collect, but if done prior to auscultation, the sounds will be altered. 3. Percussion of the abdomen would not give specific information about peptic ulcer disease. 4. Tender areas should be assessed last to prevent guarding and altering the assessment. This would include palpation, which should be done after auscultation. TEST-TAKING HINT: The word "first" requires the test taker to rank in order the interventions needed to be performed. The test taker should visualize caring for the client. This will assist the test taker in making the correct choice.

The client has end-stage liver failure secondary to alcoholic cirrhosis. Which complication indicates the client is at risk for developing hepatic encephalopathy? 1. Gastrointestinal bleeding. 2. Hypoalbuminemia. 3. Splenomegaly. 4. Hyperaldosteronism.

***1. Blood in the intestinal tract is digested as a protein, which increases serum ammonia levels and increases the risk of developing hepatic encephalopathy. 2. Decreased albumin would cause the client to develop ascites. 3. An enlarged spleen increases the rate at which RBCs, WBCs, and platelets are destroyed, which causes the client to develop anemia, leukopenia, and thrombocytopenia, but not hepatic encephalopathy. 4. An increase in aldosterone causes sodium and water retention that, in turn, causes the development of ascites and generalized edema. TEST-TAKING HINT: Some questions require the test taker to have specific knowledge to be able to identify the correct answer. This is one (1) of these questions.

The client has a large abdominal wound that has eviscerated. Which intervention should the nurse implement? 1. Apply sterile normal saline dressing. 2. Use sterile gloves to replace protruding parts. 3. Place the client in the reverse Trendelenburg position. 4. Administer intravenous antibiotic stat.

***1. Evisceration is a life-threatening condition in which the abdominal contents have protruded through the ruptured incision. The nurse must protect the bowel from the environment by placing a sterile normal saline dressing on it. The saline prevents the intestines from drying out and necrosing. 2. The nurse should not attempt to replace the protruding bowel. 3. This position places the client with the head of the bed elevated, which will make the situation worse. 4. Antibiotics will not protect the protruding bowels, which must be priority. Antibiotics will be administered at a later time to prevent infection, but this is not urgent. TEST-TAKING HINT: The test taker must understand the term "evisceration" to answer this question.

The client is in end-stage liver failure and has vitamin K deficiency. Which interventions should the nurse implement? Select all that apply. 1. Avoid rectal temperatures. 2. Use only a soft toothbrush. 3. Monitor the platelet count. 4. Use small-gauge needles. 5. Assess for asterixis.

***1. Vitamin K deficiency causes impaired coagulation; therefore rectal thermometers should be avoided to prevent bleeding. ***2. Soft toothbrushes will help prevent bleeding of the gums. ***3. Platelet count, PTT/PT, and INR should be monitored to assess coagulation status. ***4. Injections should be avoided, if at all possible, because the client is unable to clot, but if they are absolutely necessarily, the nurse should use small-gauge needles. 5. Asterixis is a flapping tremor of the hands when the arms are extended and indicates an elevated ammonia level, but it is not associated with vitamin K deficiency. TEST-TAKING HINT: The test taker must know the function of specific vitamins. Vitamin K is responsible for blood clotting. This is an alternate- type question, which requires the test taker to select all interventions that apply; the test taker should select interventions that address bleeding.

The nurse is caring for clients in an outpatient clinic. Which information should the nurse teach regarding the American Cancer Society's recommendations for the early detection of colon cancer? 1. Beginning at age 60, a digital rectal exam should be done yearly. 2. After the client reaches middle age, a yearly fecal occult test should be done. 3. At age 50, a colonoscopy and then once every five (5) to ten (10) years. 4. A flexible sigmoidoscopy should be done yearly after age 40.

1. A digital rectal exam is done to detect rectal cancer and should be started at age 40 years. 2. "Middle age" is a relative term; specific ages are used for recommendation. ****3. The American Cancer Society recommends a colonoscopy at age 50 and every five (5) to ten (10) years thereafter and a flexible sigmoidoscopy and barium enema every five (5) years. 4. A flexible sigmoidoscopy should be done at five (5)-year intervals between the colonoscopy. TEST-TAKING HINT: A digital examination is an examination that is performed by the examiner's finger and does not examine the entire colon.

The client diagnosed with gastroesophageal reflux disease (GERD) is at greater risk for which disease? 1. Hiatal hernia. 2. Gastroenteritis. 3. Esophageal cancer. 4. Gastric cancer.

1. A hiatal hernia places the client at risk for GERD; GERD does not predispose the client for developing a hiatal hernia. 2. Gastroenteritis is an inflammation of the stomach and intestine, usually caused by a virus. ****3. Barrett's esophagitis results from longterm erosion of the esophagus as a result of reflux of stomach contents secondary to GERD. This is a precursor to esophageal cancer. 4. The problems associated with GERD result from the reflux of acidic stomach contents into the esophagus, which is not a precursor to gastric cancer. TEST-TAKING HINT: The test taker may associate hiatal hernia with GERD. One (1) can be a result of the other, and this can confuse the test taker If the test taker did not have any idea of the correct answer, "3" has the word"esophageal" in it, as does the stem of the question, and therefore the test taker should select this as the correct answer.

When the nurse is conducting the initial interview, which specific data should the nurse obtain from the client who is suspected of having peptic ulcer disease? 1. History of side effects experienced from all medication. 2. Use of nonsteroidal anti-inflammatory drugs (NSAIDs). 3. Any known allergies to drugs and environmental factors. 4. Medical histories of at least three (3) generations.

1. A history of problems the client has experienced with medications is taken during the admission interview. This information does not specifically address peptic ulcer disease. **** 2. Use of NSAIDs places the client at risk for peptic ulcer disease and hemorrhage. Any client suspected of having peptic ulcer disease should be questioned specifically about the use of NSAIDs. 3. In taking the history of clients, allergies are included for safety, but this is not specific for peptic ulcer disease. 4. Information needs to be collected about past generations so that the nurse can analyze any potential health problems, but this is not specific for peptic ulcer disease. TEST-TAKING HINT: The words "specific to peptic ulcer disease" indicate that there will be appropriate data in one (1) or more of the answer options but only one (1) is specific to peptic ulcer disease. The test taker needs to read the question carefully to select the correct answer.

Which medication would the nurse expect the health-care provider to order to treat the client diagnosed with botulism secondary to eating contaminated canned goods? 1. An antidiarrheal medication. 2. An aminoglycoside antibiotic. 3. An antitoxin medication. 4. An ACE inhibitor medication.

1. Antidiarrheal medications are contraindicated with botulism because the toxin needs to be expelled from the body. A laxative may be ordered to promote removal of the toxin from the bowel. 2. Aminoglycoside antibiotics will not be ordered because there is no bacterium with botulism; it is a toxin. ****3. A botulism antitoxin neutralizes the circulating toxin and is prescribed for a client with botulism. 4. An ACE inhibitor is prescribed for a client diagnosed with cardiovascular disease. TEST-TAKING HINT: The key word in this question is "treat." Because botulism does not end in "itis, and thus is not an infection, the use of an antibiotic (option "2") can be eliminated. Antidiarrheal medication treats the diarrhea, which is a symptom of a disease, not a disease itself.

The client has been admitted to the hospital with hemorrhaging from a duodenal ulcer. Which collaborative interventions should the nurse implement? Select all that apply. 1. Perform a complete pain assessment. 2. Evaluate BP lying, sitting, and standing 3. Administer antibiotics intravenously. 4. Administer blood products. 5. Monitor intake of a soft, bland diet.

1. A pain assessment is an independent intervention that the nurse should implement frequently. 2. Evaluating blood pressure is an independent intervention that the nurse should implement. If the client is able, B/Ps should be taken lying, sitting, and standing to assess for orthostatic hypotension ***3. This is a collaborative intervention that the nurse should implement. It requires an order from the HCP. ****4. Administering blood products is collaborative, requiring an order from the HCP. 5. The diet needs an order by the health-care provider, but it would not be given to a client with a bleeding ulcer. These clients are allowed nothing by mouth until the bleeding stops. TEST-TAKING HINT: Descriptive words such as "collaborative" or "independent" can be the deciding factor when determining if an answer option is correct or incorrect. These are key words that the test taker should identify. Collaborative interventions require an HCP's order or an intervention from another healthcare discipline.

The nurse is preparing to administer an aminoglycoside antibiotic to the client just admitted with a diagnosis of acute diverticulitis. Which intervention should the nurse implement? 1. Obtain a serum trough level. 2. Ask about drug allergies. 3. Monitor the peak level. 4. Assess the vital signs.

1. A peak and trough is drawn after the client has received at least three (3) to four (4) doses of medication, not on the initial dose because the client has just been admitted. ****2. The nurse should always ask about allergies to medication when administering medications, but especially when administering antibiotics, which are notorious for allergic reactions. 3. The peak and trough would not be drawn prior to the first dose; it is ordered after multiple doses. 4. The nurse should question when to administer the medication, but there is no vital sign that would prevent the nurse from administering this medication. TEST-TAKING HINT: The test taker must read the stem closely to realize the client was just admitted and that therefore this would be the first dose of medication to be given. This would cause the test taker to eliminate "1" and "3" as possible correct answers. Both "2" and "4" are assessment data, but the test taker should ask which one will directly affect the administration of the medication.

The client diagnosed with ulcerative colitis has had an ileostomy. Which statement indicates the client needs more teaching concerning the ileostomy? 1. "My stoma should be pink and moist." 2. "I will irrigate my ileostomy every morning." 3. "If I get a red, bumpy, itchy rash I will call my HCP." 4. "I will change my pouch if it starts leaking."

1. A pink and moist stoma indicates viable tissue and adequate circulation. A purple stoma indicates necrosis. ****2. An ileostomy will drain liquid all the time and should not routinely be irrigated; only specially trained nurses are allowed to irrigate an ileostomy. A sigmoid colostomy may need daily irrigation to evacuate feces. 3. A red, bumpy, itchy rash indicates infection with the yeast Candida albicans, which should be treated with medication. 4. The ileostomy drainage has enzymes and bile salts that are irritating and harsh to the skin; therefore, the pouch should be changed if any leakage occurs. TEST-TAKING HINT: This is an "except" question and the test taker must identify which option is not a correct action for the nurse to implement. Sometimes flipping the question— "Which interventions indicate the client understands the teaching?"—can assist in identifying the correct answer.

Which client would be most likely to have the diagnosis of diverticulosis? 1. A 60-year-old male with a sedentary lifestyle. 2. A 72-year-old female with multiple childbirths. 3. A 63-year-old female with hemorrhoids. 4. A 40-year-old male with a family history of diverticulosis.

1. A sedentary lifestyle may lead to obesity and contribute to hypertension or heart disease but usually not to diverticulosis. 2. Multiple childbirths are not a risk factor for developing diverticulosis. ****3. Hemorrhoids would indicate the client has chronic constipation, which is a strong risk factor for diverticulosis. Constipation increases the intraluminal pressure in the sigmoid colon, leading to weakness in the intestinal lining, which, in turn, causes outpouchings, or diverticula. 4. A family history is not a risk factor. Having daily bowel movements and preventing constipation will decrease the chance of developing diverticulosis. TEST-TAKING HINT: The test taker must know that constipation is the leading risk factor for diverticulosis, and if the test taker knows that hemorrhoids are caused by constipation it would lead the test taker to select "3" as the correct answer.

The male client has had abdominal surgery and is now diagnosed with peritonitis. Which assessment data support the client's diagnosis of peritonitis? 1. Absent bowel sounds and potassium level of 3.9 mEq/L. 2. Abdominal cramping and hemoglobin of 14 gm/dL. 3. Profuse diarrhea and stool specimen shows Campylobacter. 4. Hard, rigid abdomen and white blood cell count 22,000 mm.

1. Absent bowel sounds would indicate a paralytic ileus, not peritonitis, and this is a normal potassium level (3.5 to 5.5 mEq/L). 2. Abdominal cramping would not make the nurse suspect peritonitis, and the hemoglobin is normal (13-17 g/dL) 3. Campylobacter is a cause of profuse diarrhea, but it does not support a diagnosis of peritonitis. ***4. A hard, rigid abdomen indicates an inflamed peritoneum (abdominal wall cavity) resulting from an infection, which results in an elevated WBC level. TEST-TAKING HINT: The "itis" of peritonitis means inflammation, and if the test taker has no idea what the answer is, an elevated WBC count should provide a key to selecting "4" as the correct answer.

The client is four (4) hours postoperative open cholecystectomy. Which data would warrant immediate intervention by the nurse? 1. Absent bowel sounds in all four (4) quadrants. 2. The T-tube with 60 mL of green drainage. 3. Urine output of 100 mL in the past three (3) hours. 4. Refusal to turn, deep breathe, and cough.

1. After abdominal surgery, it is not uncommon for bowel sounds to be absent. 2. This is a normal amount and color of drainage. 3. The minimum urine output is 30 mL/hr. ****4. Refusing to turn, deep breathe, and cough puts the client at risk for pneumonia. This client needs immediate intervention to prevent complications. TEST-TAKING HINT: The test taker should recognize normal data such as the normal urine output and normal data for postoperative clients. The test taker should apply basic concepts when answering questions. Normal or expected outcomes do not require action.

The client diagnosed with IBD is prescribed sulfasalazine (Asulfidine), a sulfonamide antibiotic. Which statement best describes the rationale for administering this medication? 1. It is administered rectally to help decrease colon inflammation. 2. This medication slows gastrointestinal motility and reduces diarrhea. 3. This medication kills the bacteria that cause the exacerbation. 4. It acts topically on the colon mucosa to decreases inflammation.

1. Asulfidine cannot be administered rectally. Corticosteroids may be administered by enema for the local effect of decreasing inflammation but minimizing the systemic effects. 2. Antidiarrheal agents slow the gastrointestinal motility and reduce diarrhea. 3. IBD is not caused by bacteria. ****4. This antibiotic is poorly absorbed from the gastrointestinal tract and acts topically on the colonic mucosa to inhibit the inflammatory process. TEST-TAKING HINT: If the test taker doesn't know the answer, then the test taker could eliminate "2" and "4" because they do not contain the word inflammation; IBD is inflammatory bowel disease.

The male client in a health-care provider's office tells the nurse that he has been experiencing "heartburn" at night that awakens him. Which assessment question should the nurse ask? 1. "How much weight have you gained recently?" 2. "What have you done to alleviate the heartburn?" 3. "Do you consume many milk and dairy products?" 4. "Have you been around anyone with a stomach virus?"

1. Clients with heartburn are frequently diagnosed as having gastroesophageal reflux disease (GERD). GERD can occasionally cause weight loss, but not weight gain. ****2. Most clients with GERD have been selfmedicating with over-the-counter medications prior to seeking advice from a health-care provider. It is important to know what the client has been using to treat the problem. 3. Milk and dairy products contain lactose, which would be important if considering lactose intolerance, but it is not important for "heartburn." 4. Heartburn is not a symptom of a viral illness TEST-TAKING HINT: Clients will use common terms such as "heartburn" to describe symptoms. The nurse must be able to interpret or clarify the meaning of terms used with the client. Part of the assessment of a symptom requires determining what aggravates and alleviates the symptom.

The nurse has administered an antibiotic, a proton pump inhibitor, and Pepto-Bismol for peptic ulcer disease secondary to H. pylori. Which data would indicate to the nurse that the medications are effective? 1. A decrease in alcohol intake. 2. Maintaining a bland diet. 3. A return to previous activities. 4. A decrease in gastric distress.

1. Decreasing the alcohol intake indicates that the client is making some lifestyle changes. 2. Maintaining a bland diet would indicate that dietary restrictions are being followed. 3. The return to previous activities would indicate that the client has not adapted to the lifestyle changes and has returned to the previous behaviors that precipitated the peptic ulcer disease. ****4. Antibiotics, proton pump inhibitors, and Pepto-Bismol are administered to decrease the irritation of the ulcerative area and cure the ulcer. A decrease in gastric distress indicates the medication is effective. TEST-TAKING HINT: To determine the effectiveness of a medication, the test taker must know why the medication is being administered. Peptic ulcer disease causes gastric distress. If gastric distress is relieved, then the medication is effective.

The nurse has received the A.M. shift report. Which client should the nurse assess first? 1. The 44-year-old client diagnosed with peptic ulcer disease who is complaining of acute epigastric pain. 2. The 74-year-old client diagnosed with acute gastroenteritis who has had four (4) diarrhea stools during the night. 3. The 65-year-old client diagnosed with inflammatory bowel disease who has a hard, rigid abdomen and elevated temperature. 4. The 15-year-old client diagnosed with food poisoning who has vomited several times during the night shift.

1. Epigastric pain is expected in a client diagnosed with peptic ulcer disease. 2. Four (4) diarrheal stools would not be unusual in a client diagnosed with gastroenteritis. ***3. A hard, rigid abdomen and an elevated temperature are abnormal in any circumstance and the nurse should assess this client first. These are clinical manifestations of peritonitis, a potentially lifethreatening condition. 4. Vomiting is expected in a client diagnosed with food poisoning. TEST-TAKING HINT: When managing clients the nurse must be able to prioritize care. Therefore, the test taker must be able to determine which client's complaints, signs, or symptoms are not expected of the disease process. Always look at the client's age because it may help the test taker determine the best answer.

The nurse is admitting a male client to a medical floor with a diagnosis of adenocarcinoma of the rectosigmoid colon. Which assessment data support this diagnosis? 1. The client reports up to 20 bloody stools per day. 2. The client states that he has a feeling of fullness after a heavy meal. 3. The client has diarrhea alternating with constipation. 4. The client complains of right lower quadrant pain with rebound tenderness. 214 MED-SURG SUCCESS Gastrointestinal

1. Frequent bloody stools are a symptom of inflammatory bowel disease (IBD). IBD is a risk factor for cancer of the colon, but the symptoms are different when the colon becomes cancerous. 2. Most people have a feeling of fullness after a heavy meal; this does not indicate cancer. ****3. The most common symptom of colon cancer is a change in bowel habits, specifically diarrhea alternating with constipation. 4. Lower right quadrant pain with rebound tenderness would indicate appendicitis. TEST-TAKING HINT: The test taker could eliminate "4" based on anatomical position. The rectosigmoid area is in the left lower quadrant.

The nurse is discussing the therapeutic diet for the client diagnosed with diverticulosis. Which meal indicates the client understands the discharge teaching? 1. Fried fish, mashed potatoes, and iced tea. 2. Ham sandwich, applesauce, and whole milk. 3. Chicken salad on whole-wheat bread and water. 4. Lettuce, tomato, and cucumber salad and coffee.

1. Fried foods increase cholesterol. Mashed potatoes do not have the peel, which is needed for increased fiber. 2. Applesauce does not have the peel, which is needed for increased fiber, and the option does not identify which type of bread; whole milk is high in fat. ****3. Chicken and whole-wheat bread are high in fiber, which is the therapeutic diet prescribed for clients with diverticulosis. An adequate intake of water helps prevent constipation. 4. Tomatoes and cucumbers both have seeds, and many health-care providers recommend that people with diverticulosis avoid seeds because of the possibility of the seeds entering the diverticulum and becoming trapped, leading to peritonitis. TEST-TAKING HINT: The test taker must know that a high-fiber diet is prescribed for diverticulosis and at least five (5) to six (6) foods that are encouraged or discouraged for the different types of diets. High-fiber foods are foods with peels (potato, apple) and whole-wheat products.

The client diagnosed with ulcerative colitis is prescribed a low-residue diet. Which meal selection indicates the client understands the diet teaching? 1. Grilled hamburger on a wheat bun and fried potatoes. 2. A chicken salad sandwich and lettuce and tomato salad. 3. Roast pork, white rice, and plain custard. 4. Fried fish, whole grain pasta, and fruit salad.

1. Fried potatoes, along with pastries and pies, should be avoided. 2. Raw vegetables should be avoided because this is roughage. ****3. A low-residue diet is a low-fiber diet. Products made of refined flour or finely milled grains, along with roasted, baked, or broiled meats, are recommended. 4. Fried foods should be avoided, and whole grain is high in fiber. Nuts and fruits with peels should be avoided. TEST-TAKING HINT: The test taker must know about therapeutic diets prescribed by healthcare providers. Remember low-residue is the same as low-fiber.

The client is in end-stage liver failure. Which gastrointestinal assessment data would the nurse expect to find when assessing the client? 1. Hypoalbuminemia and muscle wasting. 2. Oligomenorrhea and decreased body hair. 3. Clay-colored stools and hemorrhoids. 4. Dyspnea and caput medusae.

1. Hypoalbuminemia, decreased albumin, and muscle wasting are metabolic effects, not gastrointestinal effects. 2. Oligomenorrhea is no menses, which is a reproductive effect, and decreased body hair is an integumentary effect. ****3. Clay-colored stools and hemorrhoids are gastrointestinal effects of liver failure. 4. Dyspnea is a respiratory effect, and caput medusae (dilated veins around the umbilicus) is an integumentary effect, although it is on the abdomen. TEST-TAKING HINT: The adjective "gastrointestinal" is the key word that guides the test taker to select the correct answer. The test taker must rule out options that do not involve gastrointestinal symptoms. Although liver failure affects every body system, the question asks for a gastrointestinal effect.

The client diagnosed with gastroenteritis is being discharged from the emergency department. Which intervention should the nurse include in the discharge teaching? 1. If diarrhea persists for more than 96 hours, contact the physician. 2. Instruct the client to wash hands thoroughly before handling any type of food. 3. Explain the importance of decreasing steroids gradually as instructed. 4. Discuss how to collect all stool samples for the next 24 hours.

1. If the diarrhea persists more than 48 hours, notify the physician. Diarrhea for more than 96 hours could lead to metabolic acidosis, hypokalemia, and possible death. ***2. This should be done by the client at all times, but especially when the client has gastroenteritis. The bacteria in feces may be transferred to other people via food if hands are not washed properly 3. Steroids are not used in the treatment of gastroenteritis; antidiarrheal medication is usually prescribed. 4. The client may be asked to provide a stool specimen for culture, ova, parasites, and fecal leukocytes, but the client would not be asked for a 24-hour stool collection. TEST-TAKING HINT: If the test taker did not know the answer to this question, hand washing should be selected because it is the number one (1) intervention for preventing any type of contamination or nosocomial infection.

The client with a new colostomy is being discharged. Which statement made by the client indicates the need for further teaching? 1. "If I notice any skin breakdown I will call the HCP." 2. "I should drink only liquids until the colostomy starts to work." 3. "I should not take a tub bath until the HCP okays it." 4. "I should not drive or lift more than five (5) pounds."

1. If the tissue around the stoma becomes excoriated, the client will be unable to pouch the stoma adequately, resulting in discomfort and leakage. The client understands the teaching. ****2. The client should be on a regular diet, and the colostomy will have been working for several days prior to discharge. The client's statement indicates the need for further teaching. 3. Until the incision is completely healed the client should not sit in bath water because of the potential contamination of the wound by the bath water. The client understands the teaching. 4. The client has had major surgery and should limit lifting to minimal weight. The client understands the teaching. TEST-TAKING HINT: This is an abdominal surgery and all instructions for major surgery apply. This is an "except" question; therefore three (3) options would indicate the client understands the teaching.

The nurse is preparing a client diagnosed with gastroesophageal reflux disease (GERD) for surgery. Which information should be brought to the attention of the health-care provider? 1. The client's Bernstein esophageal test was positive. 2. The client's abdominal x-ray shows a hiatal hernia. 3. The client's WBC count is 14,000 mg/dL. 4. The client's hemoglobin is 13.8 mg/dL.

1. In a Bernstein's test, acid is instilled into the distal esophagus; this causes immediate heartburn for a client diagnosed with GERD. This would not warrant notifying the HCP. 2. Hiatal hernias are frequently the cause of GERD; therefore, this finding would not warrant notifying the HCP. ****3. The client's WBC is elevated, indicating a possible infection, which warrants notifying the HCP. 4. This is a normal hemoglobin result and would not warrant notifying the HCP. TEST-TAKING HINT: When the test taker is deciding when to notify a health-care provider, the answer should be data that are not normal for the disease process or that signal a potential or life-threatening complication.

The nurse writes a psychosocial problem of "risk for altered sexual functioning related to new colostomy." Which intervention should the nurse implement? 1. Tell the client that there should be no intimacy for at least three (3) months. 2. Ensure that the client and significant other are able to change the ostomy pouch. 3. Demonstrate with charts possible sexual positions for the client to assume. 4. Teach the client to protect the pouch from becoming dislodged during sex.

1. Intimacy involves more than sexual intercourse. The client can be sexually active whenever the wounds are healed sufficiently to not cause pain. 2. This is an appropriate nursing intervention for home care, but it has nothing to do with sexual activity. 3. The nurse is not a sexual counselor who would have these types of charts. The nurse should address sexuality with the client but would not be considered an expert capable of explaining the advantages and disadvantages of sexual positioning. ***4. A pouch that becomes dislodged during the sexual act would cause embarrassment for the client whose body image has already been dealt a blow. TEST-TAKING HINT: Answer option "2" does not address the issue and "3" is outside of the nurse's professional expertise. Option "1" could be eliminated because of the word "no," which is an absolute word.

Which assessment question would be priority for the nurse to ask the client diagnosed with end-stage liver failure secondary to alcoholic cirrhosis? 1. How many years have you been drinking alcohol? 2. Have you completed an advanced directive? 3. When did you have your last alcoholic drink? 4. What foods did you eat at your last meal?

1. It really doesn't matter how long the client has been drinking alcohol. The diagnosis of alcoholic cirrhosis indicates the client has probably been drinking for many years. 2. An advance directive is important for the client who is terminally ill, but it is not the priority question. ***3. The nurse must know when the client had the last alcoholic drink to be able to determine when and if the client will experience delirium tremens, the physical withdrawal from alcohol. 4. This is not a typical question asked by the nurse unless the client is malnourished, which is not information given in the stem. TEST-TAKING HINT: Because the word "alcohol" is in the stem of the question, and if the test taker had no idea what the correct answer is, the test taker should select options that have the word "alcohol" in them and look closely at options "1" and "3."

The emergency department nurse knows the client diagnosed with acute gastroenteritis understands the discharge teaching when the client makes which statement? 1. "I will probably have some leg cramps while I have gastroenteritis." 2. "I should decrease my fluid intake until the diarrhea subsides." 3. "I should reintroduce solid foods very slowly into my diet." 4. "I should only drink bottled water until the abdominal cramping stops."

1. Leg cramps could indicate hypokalemia, which is a potential complication of excessive diarrhea and should be reported to the health-care provider. 2. The client should increase the fluid intake because oral rehydration is the primary treatment for gastroenteritis to replace lost fluid as a result of diarrhea and to prevent dehydration. ***3. Reintroducing solid foods slowly, in small amounts, will allow the bowel to rest and the mucosa to return to health after acute gastroenteritis states. 4. Bottled water should be consumed when contaminated water is suspected, and an oral glucose-electrolyte solution, such as Gatorade or Pedialyte, should be recommended. TEST-TAKING HINT: Both answer options "2" and "4" refer to fluids, which should make the test taker either eliminate both of these or select from one (1) of these two (2) as the right answer.

The client with a history of peptic ulcer disease has been admitted into the hospital intensive care unit with frank gastric bleeding. Which priority intervention should the nurse implement? 1. Maintain a strict record of intake and output. 2. Insert a nasogastric tube and begin saline lavage. 3. Assist the client with keeping a detailed calorie count. 4. Provide a quite environment to promote rest.

1. Maintaining a strict record of intake and output is important to evaluate the progression of Gastrointestinal the client's condition, but it is not the most important intervention. ****2. Inserting a nasogastric tube and lavaging the stomach with saline is the most important intervention because this directly stops the bleeding. 3. A calorie count is important information that can assist in the prevention and treatment of a nutritional deficit, but this intervention does not address the client's immediate and lifethreatening problem. 4. Promoting a quiet environment aids in the reduction of stress, which can cause further bleeding, but this will not stop the bleeding. TEST-TAKING HINT: The test taker is required to rank the importance of interventions in the question. Using Maslow's Hierarchy of Needs to rank real physiologic needs first, the test taker should realize that inserting a nasogastric tube and beginning lavage is solving a circulation or fluid deficit problem.

Which statement made by the client would alert the nurse that the client may be experiencing GERD? 1. "My chest hurts when I walk up the stairs in my home." 2. "I take antacid tablets with me wherever I go." 3. "My spouse tells me I snore very loudly at night." 4. "I drink six (6) to seven (7) soft drinks every day."

1. Pain in the chest when walking up stairs would indicate angina. ****2. Frequent use of antacids indicates an acidreflux problem. 3. Snoring loudly could indicate sleep apnea, but not GERD. 4. Carbonated beverages increase stomach pressure. Six (6) to seven (7) soft drinks a day would not be tolerated by a client with GERD. TEST-TAKING HINT: The stem of the question indicates an acid problem. The drug classification of antacid or "against acid" gives the test taker a hint as to the correct answer.

The nurse is administering morning medications at 0730. Which medication would have priority? 1. A proton pump inhibitor. 2. A nonnarcotic analgesic. 3. A histamine receptor antagonist. 4. A mucosal barrier agent.

1. Proton pump inhibitors can be administered at routine dosing times, usually 0900 or after breakfast. 2. Pain medication is important, but a nonnarcotic medication, such as Tylenol, can be administered after a medication that must be timed. 3. A histamine receptor antagonist can be administered at routine dosing times. ****4. A mucosal barrier agent must be administered on an empty stomach for the medication to coat the stomach. TEST-TAKING HINT: Basic knowledge of how medications work is required to administer medications for peak effectiveness.

The charge nurse is making assignments. Staffing includes a registered nurse with five (5) years of medical-surgical experience, a newly graduated registered nurse, and two (2) unlicensed nursing assistants. Which client should be assigned to the most experienced nurse? 1. The 39-year-old client diagnosed with lower esophageal dysfunction who is complaining of pyrosis. 2. The 54-year-old client diagnosed with Barrett's esophagitis who is scheduled to have an endoscopy this morning. 3. The 46-year-old client diagnosed with gastroesophageal reflux disease who has wheezes in all five (5) lobes. 4. The 68-year-old client who is three (3) days post-op hiatal hernia and needs to be ambulated four (4) times today.

1. Pyrosis is heartburn and is expected in a client diagnosed with GERD. The new graduate can care for this client. 2. Barrett's esophagitis is a complication of GERD; new graduates can prepare a client for a diagnostic procedure. ****3. This client is exhibiting symptoms of asthma, a complication of GERD; therefore, the client should be assigned to the most experienced nurse. 4. This client can be cared for by the new graduate, and ambulating can be delegated to the unlicensed nursing assistant. TEST-TAKING HINT: The most experienced nurse should be assigned to the client whose assessment and care require more experience and knowledge about the disease process, potential complications, and medications. The adjective in the stem "most experienced" is the key to answering this question.

the client diagnosed with end-stage liver failure is admitted with hepatic encephalopathy. Which dietary restriction should be implemented by the nurse to address this complication? 1. Restrict sodium intake to 2 g/day. 2. Limit oral fluids to 1500 mL/day. 3. Decrease the daily fat intake. 4. Reduce protein intake to 60 to 80 g/day.

1. Sodium is restricted to reduce ascites and generalized edema, not for hepatic encephalopathy. 2. Fluids are calculated based on diuretic therapy, urine output, and serum electrolyte values; fluids do not affect hepatic encephalopathy. 3. A diet high in calories and moderate in fat intake is recommended to promote healing. ***4. Ammonia is a byproduct of protein metabolism and contributes to hepatic encephalopathy. Reducing protein intake should decrease ammonia levels. TEST-TAKING HINT: The test taker could eliminate options "1" and "2" based on the knowledge that sodium and water work together and address edema, not encephalopathy. The test taker's knowledge of biochemistry—protein breaks down to ammonia, carbohydrates break down to glucose, and fat breaks down to ketones—may be helpful in selecting the correct answer.

The occupational health nurse is preparing a presentation to a group of factory workers about preventing colon cancer. Which information should be included in the presentation? 1. Wear a high filtration mask when around chemicals. 2. Eat several servings of cruciferous vegetables daily. 3. Take a multiple vitamin every day. 4. Do not engage in high-risk sexual behaviors.

1. Some cancers have a higher risk of development when the client is exposed to occupationally exposed chemicals, but cancer of the colon is not one of them. ****2. Cruciferous vegetables, such as broccoli, cauliflower, and cabbage, are high in fiber. One of the risks for cancer of the colon is a high-fat, low-fiber, and high-protein diet. The longer the transit time (the time from ingestion of the food to the elimination of the waste products) the greater the chance of developing cancer of the colon 3. A multiple vitamin may improve immune system function, but it does not prevent colon cancer. 4. High-risk sexual behavior places the client at risk for sexually transmitted diseases. A history of multiple sexual partners and initial sexual experience at an early age does increase the risk for the development of cancer of the cervix in females. TEST-TAKING HINT: The colon processes waste products from eating foods, and option "2" is the only option to mention food. Therefore, "2" would be the best option to select if the test taker did not know the correct answer.

Which assessment data would indicate to the nurse that the client's gastric ulcer has perforated? 1. Complaints of sudden, sharp, substernal pain. 2. Rigid, boardlike abdomen with rebound tenderness. 3. Frequent, clay-colored, liquid stool. 4. Complaints of vague abdominal pain in the right upper quadrant.

1. Sudden sharp pain felt in the substernal area is indicative of angina or myocardial infarction. ****2. A rigid boardlike abdomen with rebound tenderness is the classic sign and symptom of peritonitis, which is a complication of a perforated gastric ulcer. 3. Clay-colored stools indicate liver disorders, such as hepatitis. 4. Clients with gallbladder disease report vague to sharp abdominal pain in the right upper quadrant. TEST-TAKING HINT: The only two (2) answer options that have the word "abdomen" are "2" or "4." Therefore, the test taker should select one (1) of these two (2) because a gastric ulcer involves the stomach.

The client diagnosed with inflammatory bowel disease has a serum potassium level of 3.4 mEq/L. Which action should the nurse implement first? 1. Notify the health-care provider. 2. Assess the client for leg cramps. 3. Request telemetry for the client. 4. Prepare to administer potassium IV.

1. The HCP needs to be notified so that potassium supplements can be ordered, but this is not the first intervention. ****2. Leg cramps are a sign of hypokalemia; hypokalemia can lead to cardiac dysrhythmias and can be life threatening. Assessment is priority for a potassium level that is just below normal level, which is 3.5 to 5.5 mEq/L. 3. Hypokalemia can lead to cardiac dysrhythmias; therefore, requesting telemetry is appropriate, but it is not the first intervention. 4. The client will need potassium to correct the hypokalemia, but it is not the first intervention. TEST-TAKING HINT: When the question asks which action should be implemented first, remember that assessment is the first step in the nursing process. If the answer option that addresses assessment is appropriate for the situation in the question, then the test taker should select it as the correct answer.

The 79-year-old client diagnosed with acute gastroenteritis is admitted to the medical unit. Which nursing task would be most appropriate for the nurse to delegate to the unlicensed nursing assistant? 1. Evaluate the client's intake and output. 2. Take the client's vital signs. 3. Change the client's intravenous solution. 4. Assess the client's perianal area.

1. The assistant can calculate the client's intake and output, but the nurse must evaluate the data to determine if it is normal for the elderly client diagnosed with acute gastroenteritis. ****2. The assistant can take the vital signs for a client who is stable; the nurse must interpret and evaluate the vital signs. 3. The assistant cannot administer medications, and IV solutions are considered to be medications. 4. The nurse cannot delegate assessment. The client may have an excoriated perianal area secondary to diarrhea; therefore, the nurse should assess the client. TEST-TAKING HINT: The nurse should not delegate any nursing task that requires judgment or assessment and cannot delegate the administration of medications. Words like "evaluate" mean the same thing as assess; therefore option "1," "3," and "4" can be eliminated.

The client is one (1) day postoperative major abdominal surgery. Which client problem is priority? 1. Impaired skin integrity. 2. Fluid and electrolyte imbalance. 3. Altered bowel elimination. 4. Altered body image.

1. The client has a surgical incision, which impairs the skin integrity, but it is not the priority because it is sutured under sterile conditions. ***2. After abdominal surgery, the body distributes fluids to the affected area as part of the healing process. These fluids are shifted from the intravascular compartment to the interstitial space, which causes potential fluid and electrolyte imbalance. 3. Bowel elimination is a problem, but after general anesthesia wears off, the bowel sounds will return and this is not a life-threatening problem. 4. Psychosocial problems are not priority over actual physiological problems. TEST-TAKING HINT: When identifying priority problems the test taker can eliminate any psychosocial problem as a potential correct answer if there are applicable physiological problems.

The nurse is preparing a client diagnosed with GERD for discharge following an esophagogastroduodenoscopy. Which statement indicates the client understands the discharge instructions? 1. "I should not eat for twenty-four (24) hours following this procedure." 2. "I can lie down whenever I want after a meal. It won't make a difference." 3. "The stomach contents won't bother my esophagus but will make me nauseous." 4. "I should avoid drinking orange juice and eating tomatoes until my esophagus heals."

1. The client is allowed to eat as soon as the gag reflex has returned. 2. An esophagogastroduodenoscopy is a diagnostic procedure, not a cure. Therefore the client still has GERD and should be instructed to stay in an upright position for two (2) to three (3) hours after eating. 3. Stomach contents are acidic and will erode the esophageal lining. ****4. Orange and tomato juices are acidic, and the client diagnosed with GERD should avoid acidic foods until the esophagus has had a chance to heal. TEST-TAKING HINT: This question assumes the test taker has knowledge of diagnostic procedures for specific disease processes.

The nurse is caring for a client diagnosed with gastroesophageal reflux disease (GERD). Which nursing interventions should be implemented? 1. Place the client prone in bed and administer nonsteroidal anti-inflammatory medications. 2. Have the client remain upright at all times and walk for 30 minutes three (3) times a week. 3. Instruct the client to maintain a right lateral side-lying position and take antacids before meals. 4. Elevate the head of the bed 30 degrees and discuss lifestyle modifications with the client.

1. The client is encouraged to lie with the head of the bed elevated, but this is difficult to achieve when on the stomach. NSAIDs inhibit prostaglandin synthesis in the stomach, which places the client at risk for developing gastric ulcers. The client is already experiencing gastric acid difficulty. 2. The client will need to lie down at some time, and walking will not help with GERD. 3. If lying on the side, the left side-lying position, not the right side, will allow less chance of reflux into the esophagus. Antacids are taken one (1) and three (3) hours after a meal. ****4. The head of the bed should be elevated to allow gravity to help in preventing reflux. Lifestyle modifications of losing weight, making dietary modifications, attempting smoking cessation, discontinuing the use of alcohol, and not stooping or bending at the waist all help to decrease reflux. TEST-TAKING HINT: Answer option "2" has an "all," which should alert the test taker that this is an incorrect response. If the test taker has no idea of the answer, lifestyle modifications are an educated guess for most chronic problems.

The nurse is planning the care of a client diagnosed with lower esophageal sphincter dysfunction. Which dietary modifications should be included in the plan of care? 1. Allow any of the client's favorite foods as long as the amount of the food is limited. 2. Have the client perform eructation exercises several times a day. 3. Eat four (4) to six (6) small meals a day and limit fluids during mealtimes. 4. Encourage the client to consume a glass of red wine with one (1) meal a day.

1. The client is instructed to avoid spicy and acidic foods and any food that produces symptoms. 2. Eructation means belching, which is a symptom of GERD. ****3. Clients should eat small, frequent meals and limit fluids with the meals to prevent reflux into the esophagus from a distended stomach. 4. Clients are encouraged to forgo all alcoholic beverages because alcohol relaxes the lower esophageal sphincter and increases the risk of reflux. TEST-TAKING HINT: The word "any" in answer option "1" should give the test taker a clue that unless there are absolutely no dietary restrictions, this is an incorrect answer. Option "2" requires knowledge of medical terminology.

The nurse is teaching the client diagnosed with diverticulosis. Which instruction should the nurse include in the teaching session? 1. Discuss the importance of drinking 1000 mL of water daily. 2. Instruct the client to exercise at least three (3) times a week. 3. Teach the client about a eating a low-residue diet. 4. Explain the need to have daily bowel movements

1. The client should drink at least 3000 mL of water daily to help prevent constipation. 2. The client should exercise daily to help prevent constipation. 3. The client should eat a high-fiber diet to help prevent constipation. ****4. The client should have regular bowel movements, preferably daily. Constipation may cause diverticulitis, which is a potentially life-threatening complication of diverticulosis. TEST-TAKING HINT: The test taker must be sure to distinguish between "osis" and "itis." diverticulosis is the condition of having small pouches in the colon, and preventing constipation is the most important action the client can take to prevent diverticulitis, inflammation of the diverticulum.

74. The client has had a liver biopsy. Which post-procedure intervention should the nurse implement? 1. Instruct the client to void immediately. 2. Keep the client NPO for eight (8) hours. 3. Place the client on the right side. 4. Monitor blood urea nitrogen (BUN) and creatinine level.

1. The client should empty the bladder immediately prior to the liver biopsy, not after the procedure. 2. Foods and fluids are usually withheld two (2) hours after the biopsy, after which the client can resume the usual diet. ***3. Direct pressure is applied to the site, and then the client is placed on the right side to maintain site pressure. 4. BUN and creatinine levels are monitored for kidney function, not liver function, and the renal system is not affected with the liver biopsy. TEST-TAKING HINT: The adjective "post-procedure" should help the test taker rule out option "1." Knowing the anatomical position of the liver should help the test taker select "3" as the correct answer. The test taker must know laboratory data for each organ, which would help rule out "4" as a possible correct answer

The client is admitted to the medical unit with a diagnosis of acute diverticulitis. Which health-care provider's order should the nurse question? 1. Insert a nasogastric tube. 2. Start IV D5W at 125 mL/hr. 3. Put client on a clear liquid diet. 4. Place client on bed rest with bathroom privileges.

1. The client will have a nasogastric tube because the client will be NPO, which will decompress the bowel and help remove hydrochloric acid. 2. Preventing dehydration is a priority with the client who is NPO. ****3. The nurse should question a clear liquid diet because the bowel must be put on total rest, which means NPO. 4. The client is in severe pain and should be on bed rest, which will help rest the bowel. TEST-TAKING HINT: This is an "except" question. Therefore the test taker must identify which answer option is incorrect for the stem. Sometimes flipping the question helps in selecting the correct answer. In this question, the test taker could ask, "Which HCP order would be expected for a client diagnosed with diverticulitis?" The option that is not expected would be the correct answer.

The client presents with a complete blockage of the large intestine from a large tumor. Which health-care provider's order would the nurse question? 1. Obtain consent for a colonoscopy and biopsy. 2. Start an IV of 0.9% saline at 125 mL/hour. 3. Administer 3 liters of Go-LYTELY. 4. Give tap water enemas until it is clear.

1. The client will need to have diagnostic tests so this is an appropriate intervention. 2. The client who has an intestinal blockage will need to be hydrated. ****3. This client has an intestinal blockage from a solid tumor blocking the colon. Although the client needs to be cleaned out for the colonoscopy, this would cause severe cramping without a reasonable benefit to the client and could cause a medical emergency. 4. Tap water enemas until it is clear would be instilling water from below the tumor to try and rid the colon of any feces in that portion of the colon and the client can expel this water. TEST-TAKING HINT: The stem states a "complete blockage," which indicates the client would need surgery. Therefore, options "1" and "2" would be appropriate for surgery. The stem asks the test taker which order would be questioned so this is an "except" question.

Which assessment data would the nurse expect to find in the client diagnosed with acute gastroenteritis? 1. Decreased gurgling sounds on auscultation of the abdominal wall. 2. A hard, firm edematous abdomen on palpation. 3. Frequent, small melena-type liquid bowel movements. 4. Bowel assessment reveals loud, rushing bowel sounds.

1. The client would have increased gurgling sounds revealing hyperactive bowel movements. 2. A hard, firm, edematous abdomen would not be expected in a client with gastroenteritis; this would indicate a possible complication and require further assessment. 3. The client would have increased liquid bowel movements, diarrhea, but would not have blood in the stool, which is the definition of melena. ****4. Borborygmi, or loud, rushing bowel sounds, indicates increased peristalsis, which occurs in clients with diarrhea and is the primary clinical manifestation in a client diagnosed with acute gastroenteritis. TEST-TAKING HINT: The test taker should realize that in an acute condition, the assessment data would be abnormal, which may help select the correct answer for some questions, However, in this question, all of the answer options are abnormal data so this will not help in this situation.

The client is two (2) hours post-colonoscopy. Which assessment data would warrant intermediate intervention by the nurse? 1. The client has a soft, nontender abdomen. 2. The client has a loose, watery stool. 3. The client has hyperactive bowel sounds. 4. The client's pulse is 104 and BP is 98/60.

1. The client's abdomen should be soft and nontender; therefore, this finding would not require immediate intervention. 2. The client had to clean the bowel prior to the colonoscopy; therefore, watery stool would be expected. 3. The client was NPO and received bowel preparation prior to the colonoscopy; therefore, hyperactive bowel sounds might occur and would not warrant immediate intervention. ****4. Bowel perforation is a potential complication of a colonoscopy. Therefore signs of hypotension—decreased BP and increased pulse—would warrant immediate intervention from the nurse. TEST-TAKING HINT: This is an "except" question. The test taker is being asked to select which data are abnormal for a procedure. The test taker should remember that any invasive procedure could possibly lead to hemorrhaging, and signs of shock should always be considered a possible correct answer.

The client is diagnosed with an acute exacerbation of ulcerative colitis. Which intervention should the nurse implement? 1. Provide a low-residue diet. 2. Monitor intravenous fluids. 3. Assess vital signs daily. 4. Administer antacids orally.

1. The client's bowel should be placed on rest and no foods or fluids should be introduced into the bowel. ****2. The client requires fluids to help prevent dehydration from diarrhea and to replace the fluid lost through normal body functioning. 3. The vital signs must be taken more often than daily in a client who is having an acute exacerbation of ulcerative colitis. 4. The client will receive anti-inflammatory and antidiarrheal medications, not antacids, which are used for gastroenteritis. TEST-TAKING HINT: "Acute exacerbation" is the key phrase in the stem of the question. The test taker should remember the concept that any lower gastrointestinal disorder should require NPO. The word "acute" should cause the test taker to eliminate any intervention that is "daily."

The client diagnosed with end-stage liver failure is admitted to the medical unit diagnosed with esophageal bleeding. The HCP inserts and inflates a triple-lumen nasogastric tube (Sengstaken-Blakemore). Which nursing action should the nurse implement for this treatment? 1. Assess the gag reflex every shift. 2. Stay with the client at all times. 3. Administer the laxative lactulose (Chronulac). 4. Monitor the client's ammonia level.

1. The client's throat is not anesthetized during the insertion of a nasogastric tube, so the gag reflex does not need to be assessed. ****2. While the balloons are inflated, the client must not be left unattended in case they become dislodged and occlude the airway. This is a safety issue. 3. This laxative is administered to decrease the ammonia level, but the question does not say that the client's ammonia level is elevated 4. Esophageal bleeding does not cause the ammonia level to be elevated. TEST-TAKING HINT: In most cases, the test taker should not select an option that contains the word "all," but in some instances, it may be the correct answer. Although the ammonia level is elevated in liver failure, the test taker must be clear as to what the question is asking. "Inflate" is the key to answering the question correctly.

Which expected outcome would be appropriate for the client scheduled to have a cholecystectomy? 1. Decreased pain management. 2. Ambulate first day postoperative. 3. No break in skin integrity. 4. Knowledge of postoperative care.

1. The expected outcome would be increased pain management for both preoperative and postoperative care. 2. Postoperative care would include ambulation. 3. Prevention of an additional break in skin integrity would be a desired postoperative outcome. The incision would be a break in skin integrity. ****4. This would be an expected outcome for the client scheduled for surgery. This indicates that preoperative teaching has been effective. TEST-TAKING HINT: The time element is important in this question. The "expected outcome" that is required is for before the client's surgery. Option "1" is wrong because of the adjective "decreased." Adjectives commonly determine the accuracy of answer options.

The client is diagnosed with gastroenteritis. Which laboratory data would warrant immediate intervention by the nurse? 1. A serum sodium level of 137 mEq/L. 2. An arterial blood gas of pH 7.37, PaO2 95, PaCO2 43, HCO3 24. 3. A serum potassium level of 3.3 mEq/L. 4. A stool sample that is positive for fecal leukocytes.

1. The normal serum sodium level is 135-145 mEq/L; therefore an intervention by the nurse is not needed. 2. These are normal arterial blood gas results; therefore, the nurse would not need to intervene. ****3. In gastroenteritis, diarrhea often results in metabolic acidosis and loss of potassium. The normal serum potassium level is 3.5-5.5 mEq/L; therefore a 3.3 mEq/L would require immediate intervention. Hypokalemia (a low potassium level) can lead to life-threatening cardiac dysrhythmias. 4. A stool specimen showing fecal leukocytes would support the diagnosis of gastroenteritis and not warrant immediate intervention by the nurse. TEST-TAKING HINT: Read the stem and make sure the test taker understands what the question is asking—in this case, "which requires immediate intervention?" Therefore the test taker is identifying an answer that is not normal for the disease process.

The client has had abdominal surgery and tells the nurse, "I felt as something just gave way in my stomach." Which action should the nurse implement first? 1. Notify the surgeon immediately. 2. Instruct the client to splint the incision. 3. Assess for serosanguineous wound drainage. 4. Administer pain medication intravenously.

1. The nurse may notify the surgeon if warranted, but that is not the first intervention. 2. The nurse should instruct the client to splint the incision when coughing but then take further action. ***3. Assessing the surgical incision is the first intervention because this may indicate the client has wound dehiscence. 4. The nurse should never administer pain medication without assessing for potential complications. TEST-TAKING HINT: The stem is asking which intervention is first. This means that all four (4) answer options could be possible actions but only one (1) is first. The test taker should use the nursing process and select the option that addresses assessment because it is the first step in the nursing process.

Which nursing interventions should be included in the care plan for the 84-year-old client diagnosed with acute gastroenteritis? Select all that apply. 1. Assess the skin turgor on the back of the client's hands. 2. Monitor the client for orthostatic hypotension. 3. Record the frequency and characteristics of sputum. 4. Use standard precautions when caring for the client. 5. Institute safety precautions when ambulating the client.

1. The nurse should assess the elderly skin turgor over the sternum because loss of subcutaneous fat associated with aging makes skin turgor assessment on the arms less reliable. ***2. Orthostatic hypotension indicates fluid volume deficit, which can occur in an elderly client who is having many episodes of diarrhea, which occurs with acute gastroenteritis. 3. The nurse should record frequency and characteristics of stool, not sputum, in the client diagnosed with gastroenteritis. ***4. Standard precautions, including wearing gloves and hand washing, help prevent the spread of the infection to others. ***5. The elderly client is at risk for orthostatic hypotension; therefore safety precautions should be instituted to ensure the client doesn't fall as a result of a decrease in blood pressure. TEST-TAKING HINT: This is an alternate-type question that requires the test taker to choose all interventions that apply. The test taker should look at each option and consider if this is an intervention for an "elderly" client. The elderly are a special population that usually have specific interventions that address the aging process no matter what the disease process.

The client complains to the nurse of unhappiness with the health-care provider. Which intervention should the nurse implement next? 1. Call the HCP and suggest he or she talk with the client. 2. Determine what about the HCP is bothering the client. 3. Notify the nursing supervisor to arrange a new HCP to take over. 4. Explain to the client that until discharge, the client will have to keep the HCP.

1. The nurse should first inform the HCP of the client's concerns and then allow the HCP and client to discuss the situation. ****2. The nurse should determine what is concerning the client. It could be a misunderstanding or a real situation where the client's care is unsafe or inadequate. 3. If a new HCP is to be arranged, it is the HCP's responsibility to arrange for another HCP to take over the care of the client. 4. The choice of HCP is ultimately the client's. If the HCP cannot arrange for another HCP, the client may have to be discharged and obtain a new health-care provider. TEST-TAKING HINT: The nurse should assess the situation; the first step in the nursing process is assessment.

The client diagnosed with Crohn's disease is crying and tells the nurse, "I can't take it anymore. I never know when I will get sick and end up here in the hospital." Which statement would be the nurse's best response? 1. "I understand how frustrating this must be for you." 2. "You must keep thinking about the good things in your life." 3. "I can see you are very upset. I'll sit down and we can talk." 4. "Are you thinking about doing anything like committing suicide?"

1. The nurse should never tell a client that they understand what they are going through. 2. This is not addressing the client's feelings. ****3. The client is crying and is expressing feelings of powerlessness; therefore the nurse should allow the client to talk. 4. The client is crying and states "I can't take it anymore," but this is not a suicidal comment or situation. TEST-TAKING HINT: There are rules that can be applied to therapeutic responses. Do not say "understand" and do not ask "why." Select an option that has some type of feeling that is being reflected in the statement.

Which assessment data indicate that the client recovering from an open cholecystectomy requires pain medication? 1. The client's pulse is 65 beats per minute. 2. The client has shallow respirations. 3. The client's bowel sounds are 20 per minute. 4. The client uses a pillow to splint when coughing.

1. The nurse would expect an increased pulse in the client who is in acute pain. ****2. Clients having abdominal pain frequently have shallow respirations. When assessing clients for pain, the nurse should discuss pain medication with any client who has shallow respirations. 3. These are normal data and would not require further action. 4. Splinting the abdomen allows the client to increase the strength of the cough by increasing comfort and would not indicate a need for pain medication. TEST-TAKING HINT: The stem asks which data would warrant pain medication. Therefore the test taker should select an answer that is not expected or is not normal for clients who are postoperative abdominal surgery.

Which assessment data support the client's diagnosis of gastric ulcer? 1. Presence of blood in the client's stool for the past month. 2. Complaints of a burning sensation that moves like a wave. 3. Sharp pain in the upper abdomen after eating a heavy meal. 4. Comparison of complaints of pain with ingestion of food and sleep.

1. The presence of blood does not specifically indicate diagnosis of an ulcer. The client could have hemorrhoids or cancer that would result in the presence of blood. 2. A wavelike burning sensation is a symptom of gastroesophageal reflux. 3. Sharp pain in the upper abdomen after eating a heavy meal is a symptom of gallbladder disease. ****4. In a client diagnosed with a gastric ulcer, pain usually occurs 30-60 minutes after eating, but not at night. In contrast, a client with a duodenal ulcer has pain during the night that is often relieved by eating food. Pain occurs 1-3 hours after meals. TEST-TAKING HINT: This question asks the test taker to identify assessment data that are specific to the disease process. Many diseases have similar symptoms, but the timing of symptoms or their location may help rule out some diseases and provide the health-care provider with a key to diagnose a specific disease— in this case, peptic ulcer disease. Nurses are usually the major source for information to the health-care team.

The client diagnosed with end-stage renal failure with ascites is scheduled for a paracentesis. Which client teaching should the nurse discuss with the client? 1. Explain that the procedure will be done in the operating room. 2. Instruct the client that a Foley catheter will have to be inserted. 3. Tell the client that vital signs will be taken frequently after the procedure. 4. Provide instructions on holding the breath when the HCP inserts the catheter.

1. The procedure is done in the client's room, with the client either seated on the side of the bed or in a chair. 2. The client should empty the bladder prior to the procedure to avoid bladder puncture, but there is no need for a Foley catheter to be inserted. ***3. The client is at risk for hypovolemia; therefore, vital signs will be assessed frequently to monitor for signs of hemorrhaging. 4. The client does not have to hold the breath when the catheter is inserted into the peritoneum; this is done when obtaining a liver biopsy. TEST-TAKING HINT: If the test taker had no idea what the answer is, knowing that vital signs are assessed after all procedures should make the test taker select this option.

The nurse has been assigned to care for a client diagnosed with peptic ulcer disease. When the nurse is evaluating care, which assessment data require further intervention? 1. Bowel sounds auscultated fifteen (15) times in one (1) minute. 2. Belching after eating a heavy and fatty meal late at night. 3. A decrease in systolic BP of 20 mm Hg from lying to sitting. 4. A decreased frequency of distress located in the epigastric region.

1. The range for normoactive bowel sounds is from five (5) to thirty-five (35) times per minute. This would require no intervention. 2. Belching after a heavy, fatty meal is a symptom of gallbladder disease. Eating late at night may cause symptoms of esophageal disorders. ****3. A decrease of 20 mm Hg in blood pressure after changing position from lying, to sitting, to standing is orthostatic hypotension. This could indicate that the client is bleeding. 4. A decrease in the quality and quantity of discomfort shows an improvement in the client's condition. This would not require further intervention. TEST-TAKING HINT: When the questions ask about further intervention, the test taker should examine the answer options for an outcome that would not be expected and would require further assessment

The client who has had an abdominal perineal resection is being discharged. Which discharge information should the nurse teach? 1. The stoma should be a white, blue, or purple color. 2. Limit ambulation to prevent the pouch from coming off. 3. Take pain medication when the pain level is at an "8." 4. Empty the pouch when it is one-third to one-half full.

1. The stoma should be light to a medium pink, the color of the intestines. A blue or purple color would indicate a lack of circulation to the stoma and is a medical emergency. 2. The stoma should be pouched securely for the client to be able to participate in normal daily activities. The client should be encouraged to ambulate to aid in recovery. 3. Pain medication should be taken before the pain level reaches a five (5). Delaying taking medication will delay the onset of pain relief and the client will not get a full benefit from the medication. ****4. The pouch should be emptied when it is one-third to one-half full to prevent the contents from becoming too heavy for the seal to hold and to prevent leakage from occurring. TEST-TAKING HINT: Normal mucosa is pink, not white, and clients are always encouraged to ambulate after surgery to prevent the complications related to immobility. Remember basic concepts when answering questions, especially postoperative nursing care.

The client diagnosed with peptic ulcer disease is admitted into the hospital. Which nursing diagnosis should the nurse include in the plan of care to observe for physiological complications? 1. Alteration in bowel elimination patterns. 2. Knowledge deficit in the causes of ulcers. 3. Inability to cope with changing family roles. 4. Potential for alteration in gastric emptying.

1. There is no indication from the question that there is a problem or potential problem with bowel elimination. 2. Knowledge deficit does not address physiological complications. 3. This client may have problems from changing roles within the family, but the question asks for potential physiological complications, not psychosocial problems. ****4. Potential for alteration in gastric emptying is caused by edema or scarring associated with peptic ulcer disease, which may cause a feeling of "fullness," vomiting of undigested food, or abdominal distention. TEST-TAKING HINT: This question asks the test taker to identify a physiological problem that identifies a complication of the disease process. Therefore, options "2" and "3" that do not address physiological problems could be eliminated.

The client admitted to the medical unit with diverticulitis is complaining of severe pain in the left lower quadrant and has an oral temperature of 100.6F. Which action should the nurse implement first? 1. Notify the health-care provider. 2. Document the findings in the chart. 3. Administer an oral antipyretic. 4. Assess the client's abdomen.

1. These are classic signs/symptoms of diverticulitis; therefore the HCP would not need to be notified. 2. These are normal findings for a client diagnosed with diverticulitis, but on admission the nurse should assess the client and document the findings in the client's chart. 3. The nurse should not administer any food or medications. ****4. The nurse should assess the client to determine if the abdomen is soft and nontender. A rigid tender abdomen may indicate peritonitis. TEST-TAKING HINT: The test taker must remember to apply the nursing process when answering test questions. Assessment is the first step in the nursing process. Although the signs/symptoms are normal and could be documented, the nurse should always assess.

The client is one (1) hour post-endoscopic retrograde cholangiopancreatogram (ERCP). Which intervention should the nurse include in the plan of care? 1. Instruct the client to cough forcefully. 2. Encourage early ambulation. 3. Assess for return of a gag reflex. 4. Administer held medications.

1. This intervention may irritate the client's throat. 2. This would not enhance safety. ****3. The endoscopic retrograde cholangiopancreatogram (ERCP) requires that an anesthetic spray be used prior to insertion of the endoscope. If medications, food, or fluid is given orally prior to the return of the gag reflex, the client may aspirate, causing pneumonia that could be fatal. 4. Medications would not be given until the gag reflex has returned. TEST-TAKING HINTS: The test taker must notice adjectives such as "endoscopic," which means the procedure includes going down the mouth; "3" is the only option that has anything to do with the mouth. If the test taker had no idea of the correct answer, selecting a distracter addressing assessment would be appropriate because assessment is the first step of the nursing process.

The client is six (6) hours postoperative open cholecystectomy and the nurse finds a large amount of red drainage on the dressing. Which intervention should the nurse implement? 1. Measure the abdominal girth. 2. Palpate the lower abdomen for a mass. 3. Turn client onto side to assess for further drainage. 4. Remove the dressing to determine the source.

1. This intervention would help further assess internal bleeding, not external bleeding. 2. This would assess the bladder, not bleeding. ****3. Turning the client to the side to assess the amount of drainage and possible bleeding is important prior to contacting the surgeon. 4. The first dressing change is usually done by the surgeon; the nurse would reinforce the dressing. TEST-TAKING HINT: The adjectives "large" and "red" indicate that the client is bleeding, and assessment is always priority when the client is having a possible complication of a surgery. Remember assessment is the first step in the nursing process.

The client is diagnosed with end-stage liver failure. The client asks the nurse, "Why is my doctor decreasing the doses of my medications?" Which statement is the nurse's best response? 1. "You are worried that your doctor has decreased the dosage." 2. "You really should ask your doctor. I am sure there is a good reason." 3. "You may have an overdose of the medication because your liver is damaged." 4. "The half-life is altered because the liver is damaged."

1. This is a therapeutic response and used to encourage the client to verbalize feelings, but it does not provide factual information. 2. This is passing the buck; the nurse should be able to answer this question. ***3. This is the main reason the HCP decreases the client's medication dose, and it is an explanation appropriate for the client. 4. This is the medical explanation as to why the medication dose is decreased, but it should not be used to explain to a layperson. TEST-TAKING HINT: The test taker should provide factual information when the client asks "why." Therefore, "1" and "2" could be eliminated as possible correct answers. Both "3" and "4" explain the rationale for decreasing the medication dose, but the nurse should answer in terms the client can understand. Would a layperson know what half-life means?

The nurse is caring for the immediate postoperative client who had a laparoscopic cholecystectomy. Which task could the nurse delegate to the unlicensed nursing assistant? 1. Check the abdominal dressings for bleeding. 2. Increase the IV fluid if the blood pressure is low. 3. Document the amount of output on the I & O sheet. 4. Listen to the breath sounds in all lobes

1. This is assessment and cannot be delegated. 2. This intervention would require analysis. ***3. This intervention would be appropriate for the nursing assistant to implement. 4. This would require assessment and cannot be delegated. TEST-TAKING HINT: The nurse cannot delegate teaching, assessing, and evaluating to a nursing assistant. The nurse cannot delegate any nursing task unless the client is stable and the task does not require judgment.

Which nursing diagnosis would be highest priority for the client who had an open cholecystectomy surgery? 1. Alteration in nutrition. 2. Alteration in skin integrity. 3. Alteration in urinary pattern. 4. Alteration in comfort.

1. This may be an appropriate client problem, but it is not priority. 2. This may be an appropriate client problem, but is not priority. 3. This may be an appropriate client problem, but is not priority. ****4. Acute pain management is the highest priority client problem after surgery because pain may indicate a life-threatening problem. TEST-TAKING HINT: When a question asks for the highest priority, the test taker should look for life-threatening problems that would be the highest priority for intervention. Pain may be expected, but it may indicate a complication.

The client is prescribed prednisone, a steroid, for an acute episode of inflammatory bowel disease. Which intervention should the nurse discuss with the client? 1. Take this medication on an empty stomach. 2. Notify the HCP if you experience a moon face. 3. Be sure to take this medication as prescribed. 4. Take the medication in the morning only

1. This medication can cause erosion of the stomach and should be taken with food. 2. A moon face is an expected side effect of prednisone. ****3. This medication must be tapered off to prevent adrenal insufficiency; therefore, the client must take this medication as prescribed. 4. There is no need to take this medication only in the morning. It is usually taken three (3) to four (4) times a day. TEST-TAKING HINT: The test taker should know that very few medications must be taken on an empty stomach, which would cause "1" to be eliminated. All medications should be taken as prescribed—don't think that the answer is too easy.

The nurse is teaching a client recovering from a laparoscopic cholecystectomy. Which statement indicates the discharge teaching was effective? 1. "I will take my lipid-lowering medicine at the same time each night." 2. "I may experience some discomfort when I eat a high-fat meal." 3. "I need someone to stay with me for about a week after surgery." 4. "I should not splint my incision when I deep breathe and cough."

1. This surgery does not require lipid-lowering medications, but eating high-fat meals may cause discomfort. ****2. After removal of the gallbladder, some clients experience abdominal discomfort when eating fatty foods. 3. Laparoscopic cholecystectomy surgeries are performed in day surgery, and clients usually do not need assistance for a week. 4. Using a pillow to splint the abdomen provides support for the incision and should be continued after discharge. TEST-TAKING HINT: When answering questions that state "teaching is effective," the test taker should look for the correct information. Basic concepts should help the test taker answer questions and, because pain often occurs after surgeries, answer option "2" would probably be a correct answer if the test taker had no idea which option is correct.

Which intervention should the nurse include when discussing ways to help prevent potential episodes of gastroenteritis from Clostridium botulism? 1. Make sure that all hamburger meat is well cooked. 2. Ensure that all dairy products are refrigerated. 3. Discuss that campers should drink only bottled water. 4. Discard all canned goods that are damaged.

1. This will help prevent gastroenteritis secondary to staphylococcal food poisoning. 2. This will help prevent gastroenteritis secondary to foods kept at room temperature, causing staphylococcal food poisoning. 3. This will help prevent gastroenteritis secondary to Escherichia coli and contaminated water. ****4. Any food that is discolored or comes from a can or jar that has been damaged or does not have a tight seal should be destroyed without tasting or touching it. TEST-TAKING HINT: The test taker should be careful with words such as "all," "only," and "never"; there are very few absolutes in the health-care field. However, when all options have these words, the test taker must find another way to choose the correct answer.

The client is admitted to the medical floor with acute diverticulitis. Which collaborative intervention would the nurse anticipate the health-care provider ordering? 1. Administer total parenteral nutrition. 2. Maintain NPO and nasogastric tube. 3. Maintain on a high-fiber diet and increase fluids. 4. Obtain consent for abdominal surgery.

1. Total parenteral nutrition is not an expected order for this client. ****2. The bowel must be put at rest. Therefore, the nurse should anticipate orders for maintaining NPO and a nasogastric tube. 3. These orders would be instituted when the client is getting better and the bowel is not inflamed. 4. Surgery is not the first consideration when the client is admitted into the hospital. TEST-TAKING HINT: Collaborative means the nurse must care for the client with another discipline and the health-care provider would have to order all of the distracters. The test taker should remember the concept that with lower gastrointestinal problems, food and fluid probably should be stopped.

The nurse identifies the client problem as "excess fluid volume" for the client in liver failure. Which short-term goal would be most appropriate for this problem? 1. The client will not gain more that two (2) kg a day. 2. The client will have no increase in abdominal girth. 3. The client's vital signs will remain within normal limits (WNL). 4. The client will receive a low-sodium diet.

1. Two (2) kg is more than four (4) pounds, which indicates severe fluid retention and is not an appropriate goal. ***2. Excess fluid volume could be secondary to portal hypertension. Therefore, no increase in abdominal girth would be an appropriate short-term goal, indicating no excess of fluid volume. 3. Vital signs are appropriate to monitor, but they do not yield specific information about fluid volume status. 4. Having the client receive a low-sodium diet does not ensure that the client will comply with the diet. The short-term goal must evaluate if the fluid volume is within normal limits. TEST-TAKING HINT: Remember that goals evaluate the interventions; therefore option "4" could be eliminated as the correct answer because it is an intervention, not a goal. Short term weight fluctuations tend to reflect fluid balance, and any weight gain in 24 hours indicates retention of fluid, which is not an appropriate goal.

The client is admitted with end-stage liver failure and is prescribed the laxative lactulose (Chronulac). Which statement indicates the client needs more teaching concerning this medication? 1. "I should have two to three soft stools a day." 2. "I must check my ammonia level daily." 3. "If I have diarrhea, I will call my doctor." 4. "I should check my stool for any blood."

1. Two to soft three stools a day indicates the medication is effective. ***2. There is no instrument that can be used at home to test daily ammonia levels. The ammonia level is a serum level that requires venipuncture and laboratory diagnostic equipment. 3. Diarrhea indicates an overdosage, possibly requiring that the dosage be decreased. The HCP would need to make this change in dosage, so the client is correct. 4. The client should check the stool for brightred blood as well as dark, tarry stool. TEST-TAKING HINT: This is an "except" question. The test taker must realize that three (3) options indicate an understanding of the teaching. If the test taker does not know the answer, notice that all the options except "2" have something to do with stool, and laxative affects the stool.

When planning the care for a client diagnosed with peptic ulcer disease, which expected outcome should the nurse include? 1. The client's pain is controlled with the use of NSAIDs. 2. The client maintains lifestyle modifications. 3. The client has no signs and symptoms of hemoptysis. 4. The client takes antacids with each meal.

1. Use of NSAIDs increases and causes problems associated with peptic ulcer disease. ***2. Maintaining lifestyle changes such as following an appropriate diet and reducing stress indicates that the client is complying with the medical teachings. Such compliance is the goal of treatment to prevent complications. 3. Hemoptysis is coughing up blood, which is not a sign or symptom of peptic ulcer disease, so not coughing up blood would not be an expected outcome for a client with peptic ulcer disease. 4. Antacids should be taken one (1) to three (3) hours after meals, not with each meal. TEST-TAKING HINT: Expected outcomes are positive completion of goals, and maintaining lifestyle modifications would be an appropriate goal for any client with any chronic illness.

The female client came to the clinic complaining of abdominal cramping and has had at least 10 episodes of diarrhea every day for the last 2 days. The client reported that she had been in Mexico on a mission trip and just returned yesterday. Which intervention should the nurse implement? 1. Instruct the client to take a cathartic laxative daily. 2. Encourage the client to drink lots of Gatorade. 3. Discuss the need to increase protein in the diet. 4. Explain that the client should weigh herself daily.

97. 1. The client would be taking antidiarrheal medication, not medications to stimulate bowel movements. ****2. The client probably has traveler's diarrhea, and oral rehydration is the preferred choice for replacing fluids lost as a result of diarrhea. An oral glucose electrolyte solution, such as Gatorade, All-Sport, or Pedialyte, is recommended. 3. The client should be encouraged to stay on liquids and eat bland foods of all three (3) food groups—carbohydrates, proteins, and fats. 4. There is no need for the client to weigh herself daily. Symptoms usually resolve within two (2) to three (3) days without complications. TEST-TAKING HINT: Be sure to note the adjectives and adverbs in the stem and the answer options, such as "cathartic" laxative and weight "daily." These words are very often important in ruling out answers and identifying the correct answer.


Ensembles d'études connexes

OSHA: Identifying Potential Hazards

View Set

module 1 practice quiz and assessments

View Set

Macedon and the Hellenistic World

View Set

Chapter 14 RNA Molecules and RNA Processing

View Set

Ormiston Chapter 4 Self Test, Ormistron Ch 3 Self Test, FRA: Understanding CF Stmts, FRA: Understanding Income Statements

View Set

U.S Government Unit 4 Vocabulary

View Set

Data Analysis with Python Week 3 Exploratory Data Analysis

View Set

Chapter 11 Special Topics in Online Privacy

View Set

Labor & Birth Process Ch. 13 workbook

View Set

Prelabor & Intrapartum Complications NCLEX

View Set